EXPONENTIAL GROWTH AND DECAY WORD PROBLEMS

In this section, we are going to see how to solve word problems on exponential growth and decay.

Before look at the problems, if you like to learn about exponential growth and decay,

please click here

Problem 1 :

David owns a chain of fast food restaurants that operated 200 stores in 1999. If the rate of increase is 8% annually, how many stores does the restaurant operate in 2007 ?

Number of years between 1999 and 2007 is 

n  =  2007 - 1999

n  =  8

No. of stores in the year 2007  =   P(1+r)ⁿ

Substitute P = 200, r = 8% or 0.08 and n = 8.

No. of stores in the year 2007  =  200(1 + 0.08) 8

No. of stores in the year 2007  =  200(1.08 ) 8

No. of stores in the year 2007  =  200(1.8509)

No. of stores in the year 2007  =  370.18

So, the number of stores in the year 2007 is about 370.

Problem 2 :

You invest $2500 in bank which pays 10% interest per year compounded continuously. What will be the value of the investment after 10 years ?

We have to use the formula given below to know the value of the investment after 3 years. 

A  =  Pe rt

Substitute 

P  =  2500

r  =  10% or 0.1

t  =  10

e  =  2.71828

Then, we have 

A = 2500(2.71828) (0.1)10

A = 6795.70

So, the value of the investment after 10 years is $6795.70.

Problem 3 :

Suppose a radio active substance decays at a rate of 3.5% per hour. What percent of substance will be left after 6 hours ?

Since the initial amount of substance is not given and the problem is based on percentage, we have to assume that the initial amount of substance is 100. 

We have to use the formula given below to find the percent of substance after 6 hours. 

A  =  P(1 + r) n

P  =  100

r  =  -3.5% or -0.035

t  =  6

(Here, the value of "r" is taken in negative sign. because the substance decays)

A  =  100(1-0.035) 6

A  =  100(0.935 ) 6

A  =  100(0.8075)

A  =  80.75

Because  the initial amount of substance is assumed as 100, the percent of substance left after 6 hours is 80.75% 

Problem 4 :

The number of bacteria in a certain culture doubles every hour. If there were 30 bacteria  present in the culture initially, how many bacteria will be present at the end of 8th hour?

Note that the number of bacteria present in the culture doubles at the end of  successive hours.

Since it grows at the constant ratio "2", the growth is based is on geometric progression. 

We have to use the formula given below to find the no. of bacteria present at the end of 8th hour. 

A  =  ab x

a  =  30

b  =  2

x  =  8

Then, we have

A  =  30(2 8 )

A  =  30(256 )

A  =  7680

So, the number of bacteria at the end of 8th hour is 7680.

Problem 5 :

A sum of money placed at compound interest doubles itself in 3 years. If interest is being compounded annually, in how many years will it amount to four times itself ?

Let "P" be the amount invested initially.  From the given information, P becomes 2P in 3 years. 

Since the investment is in compound interest, for the 4th year, the principal will be 2P.  And 2P becomes 4P (it doubles itself) in the next 3 years. 

Therefore, at the end of 6 years accumulated value will be 4P.  So, the amount deposited will amount to 4 times itself in 6 years.

Related Topics

Doubling-Time Growth Formula

Half-Life Decay Formula

Kindly mail your feedback to   [email protected]

We always appreciate your feedback.

© All rights reserved. onlinemath4all.com

  • Sat Math Practice
  • SAT Math Worksheets
  • PEMDAS Rule
  • BODMAS rule
  • GEMDAS Order of Operations
  • Math Calculators
  • Transformations of Functions
  • Order of rotational symmetry
  • Lines of symmetry
  • Compound Angles
  • Quantitative Aptitude Tricks
  • Trigonometric ratio table
  • Word Problems
  • Times Table Shortcuts
  • 10th CBSE solution
  • PSAT Math Preparation
  • Privacy Policy
  • Laws of Exponents

Recent Articles

Digital sat math problems and solutions (part - 3).

Jul 17, 24 03:39 AM

digitalsatmath3.png

Derivative Problems and Solutions (Part - 1)

Jul 16, 24 11:19 AM

Derivative Problems and Solutions (Part - 2)

Jul 16, 24 11:17 AM

Exponential Growth and Decay

Exponential growth can be amazing.

The idea: something always grows in relation to its current value, such as always doubling.

Example: If a population of rabbits doubles every month, we would have 2, then 4, then 8, 16, 32, 64, 128, 256, etc!

Amazing tree.

tree

Let us say we have this special tree.

It grows exponentially , following this formula:

Height (in mm) = e x

e is Euler's number , about 2.718

No tree could ever grow that tall. So when people say "it grows exponentially" ... just think what that means.

Growth and Decay

But sometimes things can grow (or the opposite: decay) exponentially, at least for a while .

So we have a generally useful formula:

y(t) = a × e kt

Where y(t) = value at time "t" a = value at the start k = rate of growth (when >0) or decay (when <0) t = time

Example: 2 months ago you had 3 mice, you now have 18.

Assuming the growth continues like that

Start with the formula:

We know a=3 mice, t=2 months, and right now y(2)=18 mice:

18 = 3 × e 2k

Now some algebra to solve for k :

  • The step where we used ln(e x )=x is explained at Exponents and Logarithms .
  • we could calculate k ≈ 0.896 , but it is best to keep it as k = ln(6)/2 until we do our final calculations.

We can now put k = ln(6)/2 into our formula from before:

y(t) = 3 e (ln(6)/2)t

Now let's calculate the population in 2 more months (at t=4 months):

y( 4 ) = 3 e (ln(6)/2)× 4 = 108

And in 1 year from now ( t=14 months):

y( 14 ) = 3 e (ln(6)/2)× 14 = 839,808

That's a lot of mice! I hope you will be feeding them properly.

Exponential Decay

Some things "decay" (get smaller) exponentially.

Example: Atmospheric pressure (the pressure of air around you) decreases as you go higher.

It decreases about 12% for every 1000 m: an exponential decay .

The pressure at sea level is about 1013 hPa (depending on weather).

mount everest

  • Write the formula (with its "k" value),
  • Find the pressure on the roof of the Empire State Building (381 m),
  • and at the top of Mount Everest (8848 m)
  • a (the pressure at sea level) = 1013 hPa
  • t is in meters (distance, not time, but the formula still works)
  • y(1000) is a 12% reduction on 1013 hPa = 891.44 hPa

891.44 = 1013 e k×1000

Now we know "k" we can write:

y(t) = 1013 e (ln(0.88)/1000)×t

And finally we can calculate the pressure at 381 m , and at 8848 m :

y( 381 ) = 1013 e (ln(0.88)/1000)× 381 = 965 hPa

y( 8848 ) = 1013 e (ln(0.88)/1000)× 8848 = 327 hPa

In fact pressures at the top of Mount Everest are around 337 hPa ... good calculations!

The "half life" is how long it takes for a value to halve with exponential decay.

Commonly used with radioactive decay, but it has many other applications!

Example: The half-life of caffeine in your body is about 6 hours. If you had 1 cup of coffee 9 hours ago how much is left in your system?

cup of coffee

  • a (the starting dose) = 1 cup of coffee!
  • t is in hours
  • at y(6) we have a 50% reduction (because 6 is the half life)

0.5 = 1 cup × e 6 k

Now we can write:

y(t) = 1 e (ln(0.5)/6)×t

In 6 hours:

y( 6 ) = 1 e (ln(0.5)/6)× 6 = 0.5

Which is correct as 6 hours is the half life

And in 9 hours:

y( 9 ) = 1 e (ln(0.5)/6)× 9 = 0.35

After 9 hours the amount left in your system is about 0.35 of the original amount. Have a nice sleep :)

Have a play with the Half Life of Medicine Tool to get a good understanding of this.

6.8 Exponential Growth and Decay

Learning objectives.

  • 6.8.1 Use the exponential growth model in applications, including population growth and compound interest.
  • 6.8.2 Explain the concept of doubling time.
  • 6.8.3 Use the exponential decay model in applications, including radioactive decay and Newton’s law of cooling.
  • 6.8.4 Explain the concept of half-life.

One of the most prevalent applications of exponential functions involves growth and decay models. Exponential growth and decay show up in a host of natural applications. From population growth and continuously compounded interest to radioactive decay and Newton’s law of cooling, exponential functions are ubiquitous in nature. In this section, we examine exponential growth and decay in the context of some of these applications.

Exponential Growth Model

Many systems exhibit exponential growth. These systems follow a model of the form y = y 0 e k t , y = y 0 e k t , where y 0 y 0 represents the initial state of the system and k k is a positive constant, called the growth constant . Notice that in an exponential growth model, we have

That is, the rate of growth is proportional to the current function value. This is a key feature of exponential growth. Equation 6.27 involves derivatives and is called a differential equation. We learn more about differential equations in Introduction to Differential Equations .

Rule: Exponential Growth Model

Systems that exhibit exponential growth increase according to the mathematical model

where y 0 y 0 represents the initial state of the system and k > 0 k > 0 is a constant, called the growth constant .

Population growth is a common example of exponential growth. Consider a population of bacteria, for instance. It seems plausible that the rate of population growth would be proportional to the size of the population. After all, the more bacteria there are to reproduce, the faster the population grows. Figure 6.79 and Table 6.1 represent the growth of a population of bacteria with an initial population of 200 200 bacteria and a growth constant of 0.02 . 0.02 . Notice that after only 2 2 hours ( 120 ( 120 minutes), the population is 10 10 times its original size!

Note that we are using a continuous function to model what is inherently discrete behavior. At any given time, the real-world population contains a whole number of bacteria, although the model takes on noninteger values. When using exponential growth models, we must always be careful to interpret the function values in the context of the phenomenon we are modeling.

Example 6.42

Population growth.

Consider the population of bacteria described earlier. This population grows according to the function f ( t ) = 200 e 0.02 t , f ( t ) = 200 e 0.02 t , where t is measured in minutes. How many bacteria are present in the population after 5 5 hours ( 300 ( 300 minutes)? When does the population reach 100,000 100,000 bacteria?

We have f ( t ) = 200 e 0.02 t . f ( t ) = 200 e 0.02 t . Then

There are 80,686 80,686 bacteria in the population after 5 5 hours.

To find when the population reaches 100,000 100,000 bacteria, we solve the equation

The population reaches 100,000 100,000 bacteria after 310.73 310.73 minutes.

Checkpoint 6.42

Consider a population of bacteria that grows according to the function f ( t ) = 500 e 0.05 t , f ( t ) = 500 e 0.05 t , where t t is measured in minutes. How many bacteria are present in the population after 4 hours? When does the population reach 100 100 million bacteria?

Let’s now turn our attention to a financial application: compound interest . Interest that is not compounded is called simple interest . Simple interest is paid once, at the end of the specified time period (usually 1 1 year). So, if we put $ 1000 $ 1000 in a savings account earning 2 % 2 % simple interest per year, then at the end of the year we have

Compound interest is paid multiple times per year, depending on the compounding period. Therefore, if the bank compounds the interest every 6 6 months, it credits half of the year’s interest to the account after 6 6 months. During the second half of the year, the account earns interest not only on the initial $ 1000 , $ 1000 , but also on the interest earned during the first half of the year. Mathematically speaking, at the end of the year, we have

Similarly, if the interest is compounded every 4 4 months, we have

and if the interest is compounded daily ( 365 ( 365 times per year), we have $ 1020.20 . $ 1020.20 . If we extend this concept, so that the interest is compounded continuously, after t t years we have

Now let’s manipulate this expression so that we have an exponential growth function. Recall that the number e e can be expressed as a limit:

Based on this, we want the expression inside the parentheses to have the form ( 1 + 1 / m ) . ( 1 + 1 / m ) . Let n = 0.02 m . n = 0.02 m . Note that as n → ∞ , n → ∞ , m → ∞ m → ∞ as well. Then we get

We recognize the limit inside the brackets as the number e . e . So, the balance in our bank account after t t years is given by 1000 e 0.02 t . 1000 e 0.02 t . Generalizing this concept, we see that if a bank account with an initial balance of $ P $ P earns interest at a rate of r % , r % , compounded continuously, then the balance of the account after t t years is

Example 6.43

Compound interest.

A 25-year-old student is offered an opportunity to invest some money in a retirement account that pays 5 % 5 % annual interest compounded continuously. How much does the student need to invest today to have $ 1 $ 1 million when she retires at age 65 ? 65 ? What if she could earn 6 % 6 % annual interest compounded continuously instead?

She must invest $ 135,335.28 $ 135,335.28 at 5 % 5 % interest.

If, instead, she is able to earn 6 % , 6 % , then the equation becomes

In this case, she needs to invest only $ 90,717.95 . $ 90,717.95 . This is roughly two-thirds the amount she needs to invest at 5 % . 5 % . The fact that the interest is compounded continuously greatly magnifies the effect of the 1 % 1 % increase in interest rate.

Checkpoint 6.43

Suppose instead of investing at age 25 25 , the student waits until age 35 . 35 . How much would she have to invest at 5 % ? 5 % ? At 6 % ? 6 % ?

If a quantity grows exponentially, the time it takes for the quantity to double remains constant. In other words, it takes the same amount of time for a population of bacteria to grow from 100 100 to 200 200 bacteria as it does to grow from 10,000 10,000 to 20,000 20,000 bacteria. This time is called the doubling time. To calculate the doubling time, we want to know when the quantity reaches twice its original size. So we have

If a quantity grows exponentially, the doubling time is the amount of time it takes the quantity to double. It is given by

Example 6.44

Using the doubling time.

Assume a population of fish grows exponentially. A pond is stocked initially with 500 500 fish. After 6 6 months, there are 1000 1000 fish in the pond. The owner will allow his friends and neighbors to fish on his pond after the fish population reaches 10,000 . 10,000 . When will the owner’s friends be allowed to fish?

We know it takes the population of fish 6 6 months to double in size. So, if t represents time in months, by the doubling-time formula, we have 6 = ( ln 2 ) / k . 6 = ( ln 2 ) / k . Then, k = ( ln 2 ) / 6 . k = ( ln 2 ) / 6 . Thus, the population is given by y = 500 e ( ( ln 2 ) / 6 ) t . y = 500 e ( ( ln 2 ) / 6 ) t . To figure out when the population reaches 10,000 10,000 fish, we must solve the following equation:

The owner’s friends have to wait 25.93 25.93 months (a little more than 2 2 years) to fish in the pond.

Checkpoint 6.44

Suppose it takes 9 9 months for the fish population in Example 6.44 to reach 1000 1000 fish. Under these circumstances, how long do the owner’s friends have to wait?

Exponential Decay Model

Exponential functions can also be used to model populations that shrink (from disease, for example), or chemical compounds that break down over time. We say that such systems exhibit exponential decay, rather than exponential growth. The model is nearly the same, except there is a negative sign in the exponent. Thus, for some positive constant k , k , we have y = y 0 e − k t . y = y 0 e − k t .

As with exponential growth, there is a differential equation associated with exponential decay. We have

Rule: Exponential Decay Model

Systems that exhibit exponential decay behave according to the model

where y 0 y 0 represents the initial state of the system and k > 0 k > 0 is a constant, called the decay constant .

The following figure shows a graph of a representative exponential decay function.

Let’s look at a physical application of exponential decay. Newton’s law of cooling says that an object cools at a rate proportional to the difference between the temperature of the object and the temperature of the surroundings. In other words, if T T represents the temperature of the object and T a T a represents the ambient temperature in a room, then

Note that this is not quite the right model for exponential decay. We want the derivative to be proportional to the function, and this expression has the additional T a T a term. Fortunately, we can make a change of variables that resolves this issue. Let y ( t ) = T ( t ) − T a . y ( t ) = T ( t ) − T a . Then y ′ ( t ) = T ′ ( t ) − 0 = T ′ ( t ) , y ′ ( t ) = T ′ ( t ) − 0 = T ′ ( t ) , and our equation becomes

From our previous work, we know this relationship between y and its derivative leads to exponential decay. Thus,

and we see that

where T 0 T 0 represents the initial temperature. Let’s apply this formula in the following example.

Example 6.45

Newton’s law of cooling.

According to experienced baristas, the optimal temperature to serve coffee is between 155 ° F 155 ° F and 175 ° F . 175 ° F . Suppose coffee is poured at a temperature of 200 ° F , 200 ° F , and after 2 2 minutes in a 70 ° F 70 ° F room it has cooled to 180 ° F . 180 ° F . When is the coffee first cool enough to serve? When is the coffee too cold to serve? Round answers to the nearest half minute.

Then, the model is

The coffee reaches 175 ° F 175 ° F when

The coffee can be served about 2.5 2.5 minutes after it is poured. The coffee reaches 155 ° F 155 ° F at

The coffee is too cold to be served about 5 5 minutes after it is poured.

Checkpoint 6.45

Suppose the room is warmer ( 75 ° F ) ( 75 ° F ) and, after 2 2 minutes, the coffee has cooled only to 185 ° F . 185 ° F . When is the coffee first cool enough to serve? When is the coffee be too cold to serve? Round answers to the nearest half minute.

Just as systems exhibiting exponential growth have a constant doubling time, systems exhibiting exponential decay have a constant half-life. To calculate the half-life, we want to know when the quantity reaches half its original size. Therefore, we have

Note : This is the same expression we came up with for doubling time.

If a quantity decays exponentially, the half-life is the amount of time it takes the quantity to be reduced by half. It is given by

Example 6.46

Radiocarbon dating.

One of the most common applications of an exponential decay model is carbon dating . Carbon- 14 Carbon- 14 decays (emits a radioactive particle) at a regular and consistent exponential rate. Therefore, if we know how much carbon was originally present in an object and how much carbon remains, we can determine the age of the object. The half-life of carbon- 14 carbon- 14 is approximately 5730 5730 years—meaning, after that many years, half the material has converted from the original carbon- 14 carbon- 14 to the new nonradioactive nitrogen- 14 . nitrogen- 14 . If we have 100 100 g carbon- 14 carbon- 14 today, how much is left in 50 50 years? If an artifact that originally contained 100 100 g of carbon now contains 10 10 g of carbon, how old is it? Round the answer to the nearest hundred years.

So, the model says

In 50 50 years, we have

Therefore, in 50 50 years, 99.40 99.40 g of carbon- 14 carbon- 14 remains.

To determine the age of the artifact, we must solve

The artifact is about 19,000 19,000 years old.

Checkpoint 6.46

If we have 100 100 g of carbon- 14 , carbon- 14 , how much is left after 500 500 years? If an artifact that originally contained 100 100 g of carbon now contains 20 g 20 g of carbon, how old is it? Round the answer to the nearest hundred years.

Section 6.8 Exercises

True or False ? If true, prove it. If false, find the true answer.

The doubling time for y = e c t y = e c t is ( ln ( 2 ) ) / ( ln ( c ) ) . ( ln ( 2 ) ) / ( ln ( c ) ) .

If you invest $ 500 , $ 500 , an annual rate of interest of 3 % 3 % yields more money in the first year than a 2.5 % 2.5 % continuous rate of interest.

If you leave a 100 ° C 100 ° C pot of tea at room temperature ( 25 ° C ) ( 25 ° C ) and an identical pot in the refrigerator ( 5 ° C ) , ( 5 ° C ) , with k = 0.02 , k = 0.02 , the tea in the refrigerator reaches a drinkable temperature ( 70 ° C ) ( 70 ° C ) more than 5 5 minutes before the tea at room temperature.

If given a half-life of t years, the constant k k for y = e k t y = e k t is calculated by k = ln ( 1 / 2 ) / t . k = ln ( 1 / 2 ) / t .

For the following exercises, use y = y 0 e k t . y = y 0 e k t .

If a culture of bacteria doubles in 3 3 hours, how many hours does it take to multiply by 10 ? 10 ?

If bacteria increase by a factor of 10 10 in 10 10 hours, how many hours does it take to increase by 100 ? 100 ?

How old is a skull that contains one-fifth as much radiocarbon as a modern skull? Note that the half-life of radiocarbon is 5730 5730 years.

If a relic contains 90 % 90 % as much radiocarbon as new material, can it have come from the time of Christ (approximately 2000 2000 years ago)? Note that the half-life of radiocarbon is 5730 5730 years.

The population of Cairo grew from 5 5 million to 10 10 million in 20 20 years. Use an exponential model to find when the population was 8 8 million.

The populations of New York and Los Angeles are growing at 1 % 1 % and 1.4 % 1.4 % a year, respectively. Starting from 8 8 million (New York) and 6 6 million (Los Angeles), when are the populations equal?

Suppose the value of $ 1 $ 1 in Japanese yen decreases at 2 % 2 % per year. Starting from $ 1 = ¥ 250 , $ 1 = ¥ 250 , when will $ 1 = ¥ 1 ? $ 1 = ¥ 1 ?

The effect of advertising decays exponentially. If 40 % 40 % of the population remembers a new product after 3 3 days, how long will 20 % 20 % remember it?

If y = 1000 y = 1000 at t = 3 t = 3 and y = 3000 y = 3000 at t = 4 , t = 4 , what was y 0 y 0 at t = 0 ? t = 0 ?

If y = 100 y = 100 at t = 4 t = 4 and y = 10 y = 10 at t = 8 , t = 8 , when does y = 1 ? y = 1 ?

If a bank offers annual interest of 7.5 % 7.5 % or continuous interest of 7.25 % , 7.25 % , which has a better annual yield?

What continuous interest rate has the same yield as an annual rate of 9 % ? 9 % ?

If you deposit $ 5000 $ 5000 at 8 % 8 % annual interest, how many years can you withdraw $ 500 $ 500 (starting after the first year) without running out of money?

You are trying to save $ 50,000 $ 50,000 in 20 20 years for college tuition for your child. If interest is a continuous 10 % , 10 % , how much do you need to invest initially?

You are cooling a turkey that was taken out of the oven with an internal temperature of 165 ° F . 165 ° F . After 10 10 minutes of resting the turkey in a 70 ° F 70 ° F apartment, the temperature has reached 155 ° F . 155 ° F . What is the temperature of the turkey 20 20 minutes after taking it out of the oven?

You are trying to thaw some vegetables that are at a temperature of 1 ° F . 1 ° F . To thaw vegetables safely, you must put them in the refrigerator, which has an ambient temperature of 44 ° F . 44 ° F . You check on your vegetables 2 2 hours after putting them in the refrigerator to find that they are now 12 ° F . 12 ° F . Plot the resulting temperature curve and use it to determine when the vegetables reach 33 ° F . 33 ° F .

You are an archaeologist and are given a bone that is claimed to be from a Tyrannosaurus Rex. You know these dinosaurs lived during the Cretaceous Era ( 146 ( 146 million years to 65 65 million years ago), and you find by radiocarbon dating that there is 0.000001 % 0.000001 % the amount of radiocarbon. Is this bone from the Cretaceous?

The spent fuel of a nuclear reactor contains plutonium-239, which has a half-life of 24,000 24,000 years. If 1 1 barrel containing 10 kg 10 kg of plutonium-239 is sealed, how many years must pass until only 10 g 10 g of plutonium-239 is left?

For the next set of exercises, use the following table, which features the world population by decade.

Years since 1950 Population (millions)

[T] The best-fit exponential curve to the data of the form P ( t ) = a e b t P ( t ) = a e b t is given by P ( t ) = 2686 e 0.01604 t . P ( t ) = 2686 e 0.01604 t . Use a graphing calculator to graph the data and the exponential curve together.

[T] Find and graph the derivative y ′ y ′ of your equation. Where is it increasing and what is the meaning of this increase?

[T] Find and graph the second derivative of your equation. Where is it increasing and what is the meaning of this increase?

[T] Find the predicted date when the population reaches 10 10 billion. Using your previous answers about the first and second derivatives, explain why exponential growth is unsuccessful in predicting the future.

For the next set of exercises, use the following table, which shows the population of San Francisco during the 19th century.

[T] The best-fit exponential curve to the data of the form P ( t ) = a e b t P ( t ) = a e b t is given by P ( t ) = 35.26 e 0.06407 t . P ( t ) = 35.26 e 0.06407 t . Use a graphing calculator to graph the data and the exponential curve together.

[T] Find and graph the derivative y ′ y ′ of your equation. Where is it increasing? What is the meaning of this increase? Is there a value where the increase is maximal?

[T] Find and graph the second derivative of your equation. Where is it increasing? What is the meaning of this increase?

As an Amazon Associate we earn from qualifying purchases.

This book may not be used in the training of large language models or otherwise be ingested into large language models or generative AI offerings without OpenStax's permission.

Want to cite, share, or modify this book? This book uses the Creative Commons Attribution-NonCommercial-ShareAlike License and you must attribute OpenStax.

Access for free at https://openstax.org/books/calculus-volume-1/pages/1-introduction
  • Authors: Gilbert Strang, Edwin “Jed” Herman
  • Publisher/website: OpenStax
  • Book title: Calculus Volume 1
  • Publication date: Mar 30, 2016
  • Location: Houston, Texas
  • Book URL: https://openstax.org/books/calculus-volume-1/pages/1-introduction
  • Section URL: https://openstax.org/books/calculus-volume-1/pages/6-8-exponential-growth-and-decay

© Feb 5, 2024 OpenStax. Textbook content produced by OpenStax is licensed under a Creative Commons Attribution-NonCommercial-ShareAlike License . The OpenStax name, OpenStax logo, OpenStax book covers, OpenStax CNX name, and OpenStax CNX logo are not subject to the Creative Commons license and may not be reproduced without the prior and express written consent of Rice University.

Reset password New user? Sign up

Existing user? Log in

Exponential Functions - Problem Solving

Already have an account? Log in here.

Recommended Course

Complex numbers.

The beauty of Algebra through complex numbers, fractals, and Euler’s formula.

  • Andrew Hayes

An exponential function is a function of the form \(f(x)=a \cdot b^x,\) where \(a\) and \(b\) are real numbers and \(b\) is positive. Exponential functions are used to model relationships with exponential growth or decay. Exponential growth occurs when a function's rate of change is proportional to the function's current value. Whenever an exponential function is decreasing, this is often referred to as exponential decay .

To solve problems on this page, you should be familiar with

  • rules of exponents - algebraic
  • solving exponential equations
  • graphs of exponential functions .

Growth and Decay

Problem solving - basic, problem solving - intermediate, problem solving - advanced.

Suppose that the population of rabbits increases by 1.5 times a month. When the initial population is 100, what is the approximate integer population after a year? The population after \(n\) months is given by \(100 \times 1.5^n.\) Therefore, the approximate population after a year is \[100 \times 1.5^{12} \approx 100 \times 129.75 = 12975. \ _\square \]
Suppose that the population of rabbits increases by 1.5 times a month. At the end of a month, 10 rabbits immigrate in. When the initial population is 100, what is the approximate integer population after a year? Let \(p(n)\) be the population after \(n\) months. Then \[p(n+2) = 1.5 p(n+1) + 10\] and \[p(n+1) = 1.5 p(n) + 10,\] from which we have \[p(n+2) - p(n+1) = 1.5 \big(p(n+1) - p(n)\big).\] Then the population after \(n\) months is given by \[p(0) + \big(p(1) - p(0)\big) \frac{1.5^{n} - 1}{1.5 - 1} .\] Therefore, the population after a year is given by \[\begin{align} 100 + (160 - 100) \frac{1.5^{12} - 1}{1.5 - 1} \approx& 100 + 60 \times 257.493 \\ \approx& 15550. \ _\square \end{align}\]
Suppose that the annual interest is 3 %. When the initial balance is 1,000 dollars, how many years would it take to have 10,000 dollars? The balance after \(n\) years is given by \(1000 \times 1.03^n.\) To have the balance 10,000 dollars, we need \[\begin{align} 1000 \times 1.03^n \ge& 10000 \\ 1.03^n \ge& 10\\ n \log_{10}{1.03} \ge& 1 \\ n \ge& 77.898\dots. \end{align}\] Therefore, it would take 78 years. \( _\square \)
The half-life of carbon-14 is approximately 5730 years. Humans began agriculture approximately ten thousand years ago. If we had 1 kg of carbon-14 at that moment, how much carbon-14 in grams would we have now? The weight of carbon-14 after \(n\) years is given by \(1000 \times \left( \frac{1}{2} \right)^{\frac{n}{5730}}\) in grams. Therefore, the weight after 10000 years is given by \[1000 \times \left( \frac{1}{2} \right)^{\frac{10000}{5730}} \approx 1000 \times 0.298 = 298.\] Therefore, we would have approximately 298 g. \( _\square \)

Given three numbers such that \( 0 < a < b < c < 1\), define

\[ A = a^{a}b^{b}c^{c}, \quad B = a^{a}b^{c}c^{b} , \quad C = a^{b}b^{c}c^{a}. \]

How do the values of \(A, B, C \) compare to each other?

\[\large 2^{x} = 3^{y} = 12^{z} \]

If the equation above is fulfilled for non-zero values of \(x,y,z,\) find the value of \(\frac { z(x+2y) }{ xy }\).

If \(5^x = 6^y = 30^7\), then what is the value of \( \frac{ xy}{x+y} \)?

If \(27^{x} = 64^{y} = 125^{z} = 60\), find the value of \(\large\frac{2013xyz}{xy+yz+xz}\).

\[\large f(x)=\frac{e^x+e^{-x}}{e^x-e^{-x}} \]

Suppose we define the function \(f(x) \) as above. If \(f(a)=\frac{5}{3}\) and \(f(b)=\frac{7}{5},\) what is the value of \(f(a+b)?\)

\[\large \left(1+\frac{1}{x}\right)^{x+1}=\left(1+\frac{1}{2000}\right)^{2000}\]

Given that \(x\) is an integer that satisfies the equation above, find the value of \(x\).

\[\Large a^{(a-1)^{(a-2)}} = a^{a^2-3a+2}\]

Find the sum of all positive integers \(a\) that satisfy the equation above.

Find the sum of all solutions to the equation

\[ \large (x^2+5x+5)^{x^2-10x+21}=1 .\]

\[\large |x|^{(x^2-x-2)} < 1 \]

If the solution to the inequality above is \(x\in (A,B) \), then find the value of \(A+B\).

problem solving exponential growth and decay

Master concepts like these

Learn more in our Complex Numbers course, built by experts for you.

Problem Loading...

Note Loading...

Set Loading...

Exponential Growth and Decay

Related Topics: More Lessons for High School Regents Exam Math Worksheets

High School Math based on the topics required for the Regents Exam conducted by NYSED. How to solve Exponential Growth and Decay Word Problems?

Exponential Growth Problem

Exponential Decay / Finding Half Life Find the half life of a substance that is decreasing annually by 4%.

Exponential Growth and Decay Exponential decay refers to an amount of substance decreasing exponentially. Exponential decay is a type of exponential function where instead of having a variable in the base of the function, it is in the exponent. Exponential decay and exponential growth are used in carbon dating and other real-life applications.

Mathway Calculator Widget

We welcome your feedback, comments and questions about this site or page. Please submit your feedback or enquiries via our Feedback page.

problem solving exponential growth and decay

  • HW Guidelines
  • Study Skills Quiz
  • Find Local Tutors
  • Demo MathHelp.com
  • Join MathHelp.com

Select a Course Below

  • ACCUPLACER Math
  • Math Placement Test
  • PRAXIS Math
  • + more tests
  • 5th Grade Math
  • 6th Grade Math
  • Pre-Algebra
  • College Pre-Algebra
  • Introductory Algebra
  • Intermediate Algebra
  • College Algebra

Exponential-Growth Word Problems

Log Probs Expo Growth Expo Decay

What is the formula for exponential growth?

Exponential growth word problems work off the exponential-growth formula, A  =  Pe rt , where A is the ending amount of whatever you're dealing with (for example, money sitting in an investment, or bacteria growing in a petri dish), P is the beginning amount of that same whatever, r is the growth constant, and t is time.

Content Continues Below

MathHelp.com

Exponential Growth and Decay on MathHelp.com

Exponential Growth and Decay

The exponential-growth formula A  =  Pe rt is related to the compound-interest formula , and represents the case of the interest being compounded "continuously".

Note that the particular variables used in the equation may change from one problem to another, or from one context to another, but that the structure of the equation is always the same. For instance, all of the following represent the same relationship:

A = P e r t A = P e k t Q = N e k t Q = Q 0 e k t

No matter the particular letters used, the green variable stands for the ending amount, the blue variable stands for the beginning amount, the red variable stands for the growth constant, and the purple variable stands for time. Get comfortable with this formula; you'll be seeing a lot of it.

How do you solve exponential growth problems?

To solve exponential growth word problems, you may be plugging one value into the exponential-growth equation, and solving for the required result. But you may need to solve two problems in one, where you use, say, the doubling-time information to find the growth constant (probably by solving the exponential equation by using logarithms), and then using that value to find whatever the exercise requested.

What is an example of an exponential growth problem?

  • A biologist is researching a newly-discovered species of bacteria. At time t  = 0 hours, he puts one hundred bacteria into what he has determined to be a favorable growth medium. Six hours later, he measures 450 bacteria. Assuming exponential growth, what is the growth constant k for the bacteria? (Round k to two decimal places.)

For this exercise, the units on time t will be hours, because the growth is being measured in terms of hours.

The beginning amount P is the amount at time t  = 0 , so, for this problem, P  = 100 .

The ending amount is A  = 450 at t  = 6 hours.

The only variable I don't have a value for is the growth constant k , which also happens to be what I'm looking for. So I'll plug all the known values into the exponential-growth formula, and then solve for the growth constant:

450 = 100 e 6 k

4.5 = e 6 k

ln(4.5) = 6 k

ln(4.5) / 6 = k = 0.250679566129...

They want me to round my decimal value (found by punching keys on my calculator) to two decimal places. So my answer is:

Many math classes, math books, and math instructors leave off the units for the growth and decay rates. However, if you see this topic again in chemistry or physics, you will probably be expected to use proper units ("growth-decay constant ÷ time"). If I had included this information in my solution above, my answer would have been " k  = 0.25 /hour". I doubt that your math class will even mention this, let alone require that you include this. Still, it's not a bad idea to get into the habit now of checking and reporting your units.

Note that the constant was positive, because it was a growth constant. If I had come up with a negative value for the growth constant, then I would have known to check my work to find my error(s).

  • A certain type of bacteria, given a favorable growth medium, doubles in population every 6.5 hours. Given that there were approximately 100 bacteria to start with, how many bacteria will there be in a day and a half?

In this problem, I know that time t will be in hours, because they gave me growth in terms of hours. As a result, I'll convert "a day and a half" to "thirty-six hours", so my units match.

I know that the starting population is P  = 100 , and I need to find A at time t  = 36 .

Advertisement

But what is the growth constant k ? And why do they tell me what the doubling time is?

They gave me the doubling time because I can use this to find the growth constant k . Then, once I have this constant, I can go on to answer the actual question.

So this exercise actually has two unknowns, the growth constant k and the ending amount A . I can use the doubling time to find the growth constant, at which point the only remaining value will be the ending amount, which is what they actually asked for. So first I'll find the constant.

If the initial population were, say, 100 , then, in 6.5 hours (being the specified doubling time), the population would be 200 . (It doesn't matter what starting value I pick for this part of my solution, as long as my ending value is twice as much. It's the "twice as much" that matters here, more than the number that it's twice of.) I'll set this up and solve for k :

200 = 100 e 6.5 k

2 = e 6.5 k

At this point, I need to use logs to solve:

ln(2) = 6.5 k

ln(2) / 6.5 = k

I could simplify this to a decimal approximation, but I won't, because I don't want to introduce round-off error if I can avoid it. So, for now, the growth constant will remain this "exact" value. (I might want to check this value quickly in my calculator, to make sure that this growth constant is positive, as it should be. If I have a negative value at this stage, I need to go back and check my work.)

Now that I have the growth constant, I can answer the actual question, which was "How many bacteria will there be in thirty-six hours?" This means using 100 for P , 36 for t , and the above expression for k . I plug these values into the formula, and then I simplify to find A :

A = 100 e 36(ln(2)/6.5) = 4647.75313957...

I will take the luxury of assuming that they don't want fractional bacteria (being the 0.7313957... part), so I'll round to the nearest whole number.

about 4648 bacteria

You can do a rough check of this answer, using the fact that exponential processes involve doubling times. The doubling time in this case is 6.5 hours, or between 6 and 7 hours.

If the bacteria doubled every six hours, then there would be 200 in six hours, 400 in twelve hours, 800 in eighteen hours, 1600 in twenty-four hours, 3200 in thirty hours, and 6400 in thirty-six hours.

If the bacteria doubled every seven hours, then there would be 200 in seven hours, 400 in fourteen hours, 800 in twenty-one hours, 1600 in twenty-eight hours, and 3200 in thirty-five hours.

The answer I got above, 4678 bacteria in thirty-six hours, fits nicely between these two estimates.

Algebra Tutors

Note: When you are given a nice, neat doubling time, another method for solving the exercise is to use a base of 2 . First, figure out how many doubling-times that you've been given. In the above case, this would start by noting that "a day and a half" is 36 hours, so we have:

36 ÷ 6.5 = 72/13

Use this as the power on 2 :

100 × 2 (72/13) = 4647.75314...

Not all algebra classes cover this method. If you're required to use the first method for every exercise of this type, then do so (in order to get the full points). Otherwise, this base- 2 trick can be a time-saver. And, yes, you'd use a base of 3 if you'd been given a tripling-time, a base of 4 for a quadrupling-time, etc.

URL: https://www.purplemath.com/modules/expoprob2.htm.htm

Page 1 Page 2 Page 3

Standardized Test Prep

College math, homeschool math, share this page.

  • Terms of Use
  • Privacy / Cookies
  • About Purplemath
  • About the Author
  • Tutoring from PM
  • Advertising
  • Linking to PM
  • Site licencing

Visit Our Profiles

problem solving exponential growth and decay

Solving Exponential Growth and Decay Problems

This section presents a variety of problems, utilizing ideas and techniques covered in:

  • Exponential Growth and Decay: Introduction
  • Exponential Growth and Decay: Relative Growth Rate
  • Solving Exponential Equations
  • Solving Logarithmic Equations

Once you've recognized exponential behavior (equal changes in the input cause the output to be multiplied by a constant) then you can always use $\,P(t) = P_{0}{\text{e}}^{rt}\,$ as your model. However, it is often more natural and easier to use a different base.

Initial Population and Relative Growth Rate Given; Find a Future Population Size

  • Alternative solution (easiest)
  • More alternative solutions

(Equal Change,Constant Multiplier) Pair Known

Example:   Suppose the population of a city was $\,680{,}000\,$ in $\,1992$; the relative growth rate is $\,12\%\,$ per year. What is the population $\,t\,$ years after $\,1992$? In the year $\,2000$?

Choose $\,t = 0\,.$ Here, the year corresponding to the earliest known population size is chosen.

Since the given relative growth rate is ‘$\,12\%\,$ per year’, you need to use units of ‘year’ for time when using the model $\,P(t) = P_{0}{\text{e}}^{rt}\,.$

Let $\,P(t)\,$ have units of thousands.

For example, $\,P(t) = 680\,$ means that the population at time $\,t\,$ is $\,680{,}000\,.$

Choose the units for the output. By using units of thousands, you can work with smaller numbers.

Let $\,P(t) = P_{0}{\text{e}}^{rt}\,,$ where:

  • $\,r = \text{the relative growth rate} = 12\% = 0.12\,$ per year
  • $P_{0} = \text{the population at time zero} = 680\,$

Thus, $\,P(t) = 680{\text{e}}^{0.12t}\,$ gives the population $\,t\,$ years after $\,1992\,.$

Whenever you see the phrase relative growth rate , continuous growth rate , or exponential growth rate , you know you're dealing with exponential behavior.

As discussed in Exponential Growth and Decay: Introduction , all exponential growth/decay problems can be modeled using $\,P(t) = P_{0}{\text{e}}^{rt}\,,$ where $\,r\,$ is the relative growth rate.

The year $\,2000\,$ corresponds to: $\,t = 2000-1992 = 8\,$ $P(8) = 680{\text{e}}^{(0.12)(8)} \approx 1775.954\,$

In the year $\,2000\,,$ the population will be about $\,1{,}775{,}954\,$ people.

Be careful with units! Here, $\,P(8) \approx 1775.954\,$ has units of thousands . Thus, the output of $\,1775.954\,$ from the function $\,P\,$ corresponds to a population of $\,(1775.954)(1000) = 1{,}775{,}954\,.$

Be Careful! These are not the same thing!

‘A relative growth rate of $\,12\%\,$ per year’ ‘Growing at the rate of $\,12\%\,$ per year’ (an annual growth rate)

As the next example shows, an annual growth rate of $\,12\%\,$ corresponds to a relative growth rate of approximately $\,11.33\%\,$ per year.

Initial Population and Annual Growth Rate Given; Find a Future Population Size

Example:   Suppose the population of a city was $\,680{,}000\,$ in $\,1992$; it is growing at the rate of $\,12\%\,$ per year. What is the population $\,t\,$ years after $\,1992$? In the year $\,2000$?

Recognize exponential growth: equal changes in the input (every year) cause the output to be multiplied by a constant ($\,1.12\,$).

  • We must find the relative growth rate, $\,r\,$

A growth rate of $\,12\%\,$ per year corresponds to a yearly multiplier of $\,1.12\,.$

Once exponential behavior has been identified, you can always use the model $\,P(t) = P_{0}{\text{e}}^{rt}\,.$ However, it is not always easiest.

Exponential behavior is defined by equal changes in the input causing the output to be multiplied by a constant . Whenever an ( equal change , constant multiplier ) pair is known, then there is a more natural exponential model to use, as shown below.

Use $\,P(1) = (1.12)P_{0}\,$ to solve for $\,r\,$: $\overbrace{P_{0}\,{\text{e}}^{r\cdot 1}}^{P(1)} = 1.12P_{0}$ ${\text{e}}^r = 1.12$   (divide by $\,P_{0}\,$) $r = \ln 1.12$ Thus: $\,P(t) = 680{\text{e}}^{t\ln 1.12}\,$ $\,P(8) = 680{\text{e}}^{8\cdot \ln 1.12} \approx 1683.655\,$

In the year $\,2000\,,$ the population will be about $\,1{,}683{,}655\,$ people. (An annual growth rate of $\,12\%\,$ is slightly slower than a relative growth rate of $\,12\%\,.$ )

Similarly, an annual growth rate of $\,g\,$ corresponds to a relative growth rate of $\,\ln(1 + g)\,.$

The graph below shows the annual growth rate in blue, and the relative growth rate in red. For growth rates up to about $\,5\%\,$ they are almost indistinguishable. (Note: the horizontal axis is $\,g\,.$)

annual growth rate versus relative growth rate

$\,y = g\,$ (blue) $\,y = \ln (1 + g)\,$ (red)

Alternative Solution (Easiest)

Let $\,t = 0\,$ in $\,1992\,,$ and measure $\,t\,$ in years.

Then: $\,P(t) = 680(1.12)^t\,$ $\,P(8) = 680(1.12)^8 \approx 1683.655\,$ (Of course, this is the same answer as above!) Much easier!

Looking at a few years of population, a pattern quickly emerges:

$t = 0$$P(0) = 680 = 680t^0$
$t = 1$$P(1) = 680(1.12) = 680(1.12)^1$
$t = 2$$P(2) = 680(1.12)^2$
$t$$P(t) = 680(1.12)^t$

So, even though you always have the option of using the exponential model $\,P(t) = P_{0}{\text{e}}^{rt}\,,$ it is not always in your best interest to do so!

More Alternative Solutions

Different choices (for time zero, for input/output units) will cause the solution steps to look different. As long as you're consistent with your choices, though, you'll always end up at exactly the same place!

To illustrate, here are a couple more ways of doing the previous problem. These are not recommended choices—they just illustrate possibilities.

The problem is repeated here for your convenience: Suppose the population of a city was $\,680{,}000\,$ in $\,1992$; it is growing at the rate of $\,12\%\,$ per year. What is the population $\,t\,$ years after $\,1992$? In the year $\,2000$?

(First Alternative Solution: Not Recommended!)

Let $\,t = 0\,$ in $\,2000\,,$ and measure $\,t\,$ in years.

Let $\,P(t) = P_{0}{\text{e}}^{rt}\,$; both $\,P_{0}\,$ and $\,r\,$ must be found.

Since $P(1) = (1.12)P(0)$:

Since $P(-8) = 680{,}000\,$:

(Second Alternative Solution: Not Recommended!)

Let $\,t = 0\,$ in $\,1992\,,$ and measure $\,t\,$ in weeks.

Let $\,P(t)\,$ have units of hundred-thousands.

Then $\,P(t) = 6.8{\text{e}}^{rt}\,$; $\,r\,$ must be found.

Since $P(52) = (1.12)P(0)\,$:

Therefore, the population in the year $\,2000\,$ is:

Yuck!   These are certainly not good choices to make for this problem!

Exponential behavior is defined by equal changes in the input causing the output to be multiplied by a constant . Whenever an ( equal change , constant multiplier ) pair is known, then there is a natural exponential model to use, as discussed next.

Suppose that a time change of $\,\Delta t\,$ (read as ‘delta $\,t\,$’) causes the output to successively be multiplied by a constant $\,b\,.$ Thus, we are considering the ( equal change , constant multiplier ) pair to be $\,(\Delta t,b)\,.$

Letting $\,P_{0}\,$ denote the starting amount and $\,P(t)\,$ the amount at time $\,t\,,$ a pattern quickly emerges:

Starting amount:  amount at $\,t = 0$ $P(0) = P_{0} = P_{0}b^0$
Amount at time $\Delta t$

(one time period elapses)

Prior amount gets multiplied by $\,b$ $\begin{align} P(\Delta t) &= b\cdot P_{0}b^0\cr &= P_{0}b^1 \end{align}$
Amount at time $2\Delta t$

(another time period elapses)
Prior amount gets multiplied by $\,b$ $\begin{align} P(2\Delta t) &= b\cdot P_{0}b^1\cr &= P_{0}b^2 \end{align}$
Amount at time $3\Delta t$

(another time period elapses)
Prior amount gets multiplied by $\,b\,$ $\begin{align} P(3\Delta t) &= b\cdot P_{0}b^2\cr &= P_{0}b^3 \end{align}$

In general, we have $\,P(\color{red}{k\Delta t}) = P_{0}b^{k}\,.$ But, what is the amount at an arbitrary time $\,t\,$?

Let $\,t = \color{red}{k\Delta t}\,,$ so that $k = t/\Delta t\,.$ Then we have:

is an exponential model corresponding to an ( equal change , constant multiplier ) pair $\,(\Delta t,b)\,$ and initial amount $\,P_{0}$

  • The constant multiplier becomes the base of the exponential function.
  • In the exponent, time gets divided by the ‘equal change’.

Here are examples:

First Example

Suppose something doubles every $\,3\,$ seconds. ‘Doubling’ is multiplying by $\,2\,.$

Then, the ( equal change , constant multiplier ) pair is $\,(3,2)\,.$

The model is

Second Example

Suppose something gets halved every $\,7\,$ minutes. ‘Halving’ is multiplying by $\,0.5\,.$

Then, the ( equal change , constant multiplier ) pair is $\,(7,0.5)\,.$

A variety of other names are possible:

Non-Uniqueness of ( equal change , constant multiplier  ) Pair

Note:   An ( equal change , constant multiplier ) pair is not unique! If $\,(\Delta t,b)\,$ is such a pair, then so is $\displaystyle\,(k\Delta t,b^k)\,$ for every nonzero real number $\,k\,,$ since: $$ \begin{align} P(t)\ &\cssId{s188}{= P_{0}(b^k)^{\frac{t}{k\Delta t}}}\cr\cr &\cssId{s189}{= P_{0}\,b^{k\,\cdot\,\frac{t}{k\Delta t}}}\cr\cr &\cssId{s190}{= P_{0}\,b^{t/\Delta t}} \end{align} $$

Concept Practice

Exponential Growth And Decay

Exponential growth and decay apply to physical quantities which change in value or form in a rapid manner. The change can be measured using the concept of exponential growth and exponential decay, and the new obtained quantity can be obtained from the existing quantity. The formulas of exponential growth and decay are f(x) = a(1 + r) t , and f(x) = a(1 - r) t respectively.

Let us learn more about exponential growth and decay, the formula, applications, with the help of examples, FAQs.

1.
2.
3.
4.
5.
6.

What Is Exponential Growth And Decay?

Exponential growth and decay apply to quantities which change rapidly. Exponential growth and decay have been derived from the concept of geometric progression. Quantities that do not change as constant but change in an exponential manner can be termed as having an exponential growth or exponential decay.

The simplest representation of exponential growth and decay is the formula ab x , where 'a' is the initial quantity, 'b' is the growth factor which is similar to the common ratio of the geometric progression, and 'x' in the time steps for multiplying the growth factor. For exponential growth, the value of b is greater than 1 (b > 1), and for exponential decay, the value of b is lesser than 1 (b < 1).

Exponential growth finds applications in studying bacterial growth, population increase, money growth schemes. Exponential decay refers to a rapid decrease in a quantity over a period of time. The exponential decay can be used to find food decay, half-life, radioactive decay. The formulas of exponential growth and decay are as presented below.

Exponential Growth And Decay

Exponential growth uses a factor 'r' which is the rate of growth. Here the r-value lies between 0 and 1 (0 < r < 1). The term (1 + r) can be taken as the growth factor. And 't' is the time steps which is the number of times the growth factor is to be multiplied. The value of 't' can be a whole number or a decimal number. For exponential decay, the growth factor is (1 - r), which has a value lesser than 1.

Formulas of Exponential Growth And Decay

The exponential growth and decay have different interpretations of the formulas which are interrelated and can be interpreted differently. The below table shows the three different formulas of exponential growth and decay.

f(x) = ab f(x) = ab
f(x) = a(1 + r) f(x) = a(1 - r)
P = P e P = P e

In the above formulas the 'a' or P o is the initial quantity of the substance. Further for exponential growth b = 1 + r = e k and for exponential decay we have b = 1 - r = e -k .

Applications of Exponential Growth And Decay

The concept of exponential growth and decay can be observed in numerous day-to-day scientific and industrial activities. Let us check a few important applications of exponential growth and decay.

  • Bacterial Growth: The initial bacterial growth can be observed in numerous communicable diseases. The recent Covid-19 is a quick example of exponential growth where the disease is highly communicable and is transmitted from one to many, and then to many more people. Bacteria and viruses are growing in an exponential manner, and the initial bacteria grows at an exponential rate. The initial bacterial if taken to double every second would grow in numbers such as 1, 2, 4, 8, 16, 32, 64, 128, 256, 512... With this, we can observe the manner of exponential growth in bacteria.
  • Nuclear Chain Reactions: The nuclear chain reactions can be broadly classified as nuclear fission and nuclear fission reactions. The concept of nuclear fusion can be linked with exponential growth and the concept of nuclear fission can be linked with exponential decay. Nuclear fusion is a reaction in which two or more atoms combine to form a larger atom and this kind of reaction can be observed in the core of the sun. Nuclear fission is a kind of exponential decay that can be observed in radioactive material, in which the initial quantity decomposes and we have a smaller quantity by the end of the observed time period.
  • Feedback: The concept of feedback, more so of customer feedback grows in an exponential manner. This can be observed more so in negative feedback. The bad functioning of the product if experienced by a customer is shared by this customer to another person, who in turn shares it with two or more people, and each of those people again shares it with two or more people. The medium of the internet helps for this easy sharing of feedback which flows exponentially to a larger customer segment. Thus the feedback in this current day of the internet is conveyed at a rapid pace.
  • Processing Power of Computers: The processing and storage power has now increased exponentially. Earlier the data storage was only in MB, which has now grown to GB and TB exponentially. The slow growth in the computer hardware and storage system in the initial 1970s and 1980s has now grown exponentially. The initial floppy disks and hard disk drives have now been replaced with cloud servers which can be easily assessed by the user through an internet-connected mobile device.
  • Food Degradation: Food degradation can also be understood as a case of exponent growth and decay. The food remains good for a certain amount of time and then it degenerates exponentially. The food is slightly stale initially, and then it stales rapidly until we discover that it has spoilt completely. This is also a typical case of exponential decay, where once the decay process initiates, then it decays at a rapid pace until the entire food quantity is completely stale.
  • Aging of Human Beings: The aging process of humans or any living being at the ending part of the lifetime follows an exponential decay process. The person remains hale and healthy for a normal course of life for about 60 years. This is the same reason that for many jobs the retirement age is set as 60 years. After this, the aging process is so rapid that it affects the body at an exponential rate. The degradation in the quality of life seen in the initial days is further degraded drastically in the later years. This could be because of the advancement of certain existing diseases or the malfunctioning of certain organs.
  • Internet Content: The internet is exploding with information. In the initial stages of the internet, google had to collect useful information and add it to the net. But with time the internet users started adding information to the internet, and now the amount of information now available on the internet is mind-boggling. Also now the application of artificial intelligence algorithms helps build the content in an exponential manner. The content is generated thousands and millions of times in a short span of time.

☛ Related Topics

  • Exponent Rules
  • Decay Formula
  • Negative Exponents
  • Exponential to Log Form
  • Log to Exponential Form
  • Exponential Decay Formula

Examples on Exponential Growth And Decay

Example 1: What is the amount received from the investment fund after 2 years, if $.100,000 were invested at the compounding rate of 5% per every quarter?

The invested principal is a = $100,000, the rate of compounding growth is r = 5% = 0.05 per quarter.

The time period is 2 years, and there are 4 quarters in a year, and we have t = 8.

Applying the concepts of exponential growth and decay we have the following expressions for exponential growth.

f(x) = a(1 + r) t

f(x) = 100,000(1 + 0.05) 8

f(x) = 1,00,000(1.05) 8 = 100,000 × 1.47745544 = 147745.44

Therefore an amount of $1,47, 746 is received after a period of 2 years.

Example 2: The radioactive material of thorium decays at the rate of 8% per minute. What part of 10 grams of thorium would be remaining after 5 minutes?

The given initial quantity of thorium is a = 10grams, the rate of decay per minute is r = 8% = 8/100 = 0.08, and the time steps t = 5.

Here we can apply the concepts of exponential growth and decay, and the exponential decay formula for the decay of thorium is as follows.

f(x) = a(1 - r) t

f (x) = 10(1 - 0.08) 5 = 10(0.92) 5 = 6.5908

Therefore a quantity of 6.6 grams of thorium remains after 5 minutes.

go to slide go to slide

problem solving exponential growth and decay

Book a Free Trial Class

Practice Questions on Exponential Growth And Decay

Faqs on exponential growth and decay, what is exponential growth and decay in maths.

Exponential growth and decay in maths applies to the calculation of rapidly changing quantity. There are numerous quantities and values in nature, industry, business which change rapidly with time. Compound internet, decpreciation, growth of virus, spoiling of perishable foods, are some of the real life examples which need to be calculated using a math formula. The exponential growth and decay gives the required needed calculations using the formulas f(x) = a(1 + r) t , and f(x) = a(1 - r) t . Here a is the initial quantity, r is the growth or decay constant, and t is the time period or the time factor.

What Is The Formula For Exponential Growth And Decay?

There are three types of formulas that are used for computing exponential growth and decay. The three formulas are as follows.

  • f(x) = ab x for exponential growth and f(x) = ab -x for exponential decay. Here 'a' is the initial quantity, 'b' is the growth or decay factor, and 'x' is the time step.
  • f(x) = a(1 + r) t , and f(x) = a(1 - r) t are for exponential growth and exponential decay respectively. Here 'r' is the growth or decay factor and has a value between 0 and 1, (0 < r < 1).
  • P = P o e kt , P = P o e -kt are for formulas of exponential growth and decay. Here P o is the initial quantity, P is the obtained quantity, e is the exponential factor, and k is the growth or decay constant.

What Is The Difference Between Exponential Growth And Exponential Decay?

Exponential growth refers to an increase of the resultant quantity for a given quantity, and exponential decay refers to the decrease of the resultant quantity for a given quantity. The exponential growth and decay both need the initial quantity, the time period and the decay or growth constant to find the resultant quantity. Exponetial growth finds use in finance, medicine, biology, and exponential decay find use to find the depreciation of an asset, to find the expiry date of a manufactured item.

What Is The Use Of Exponential Growth And Decay In Maths?

The exponential growth and decay can be used to calculate the resultant quantity after a process of exponential growth or exponetial decay. From the given initial quantity, and the rate of growth or decay we can easily compute the resultant quantity. We can calculate the exponent growth and decay using f(x) = a(1 + r) t , and f(x) = a(1 - r) t .

What Are The Applications Of Exponential Growth And Decay In Daily Life?

The exponential growth and decay have numerous applications in our day-to-day life. The biological world has numerous examples of diseases and their spread, micro organisms, virus and their growth, which needs to be computed. Further we find numerous examples of exponential growth in finance, business, the internet, consumer behavior.

  • Skip to navigation (Press Enter)
  • Skip to main content (Press Enter)

Math Insight

Exponential growth and decay: a differential equation.

This little section is a tiny introduction to a very important subject and bunch of ideas: solving differential equations . We'll just look at the simplest possible example of this.

The general idea is that, instead of solving equations to find unknown numbers , we might solve equations to find unknown functions . There are many possibilities for what this might mean, but one is that we have an unknown function $y$ of $x$ and are given that $y$ and its derivative $y'$ (with respect to $x$) satisfy a relation $$y'=ky$$ where $k$ is some constant. Such a relation between an unknown function and its derivative (or derivatives ) is what is called a differential equation . Many basic ‘physical principles’ can be written in such terms, using ‘time’ $t$ as the independent variable.

Having been taking derivatives of exponential functions, a person might remember that the function $f(t)=e^{kt}$ has exactly this property: $${d\over dt}e^{kt}=k\cdot e^{kt}$$ For that matter, any constant multiple of this function has the same property: $${d\over dt}(c\cdot e^{kt})=k\cdot c\cdot e^{kt}$$ And it turns out that these really are all the possible solutions to this differential equation.

There is a certain buzz-phrase which is supposed to alert a person to the occurrence of this little story: if a function $f$ has exponential growth or exponential decay then that is taken to mean that $f$ can be written in the form $$f(t)=c\cdot e^{kt}$$ If the constant $k$ is positive it has exponential growth and if $k$ is negative then it has exponential decay .

Since we've described all the solutions to this equation, what questions remain to ask about this kind of thing? Well, the usual scenario is that some story problem will give you information in a way that requires you to take some trouble in order to determine the constants $c,k$ . And, in case you were wondering where you get to take a derivative here, the answer is that you don't really: all the ‘calculus work’ was done at the point where we granted ourselves that all solutions to that differential equation are given in the form $f(t)=ce^{kt}$.

First to look at some general ideas about determining the constants before getting embroiled in story problems: One simple observation is that $$c=f(0)$$ that is, that the constant $c$ is the value of the function at time $t=0$. This is true simply because $$f(0)=ce^{k \cdot 0}=ce^{0}=c\cdot 1=c$$ from properties of the exponential function.

More generally, suppose we know the values of the function at two different times: $$y_1=ce^{kt_1}$$ $$y_2=ce^{kt_2}$$ Even though we certainly do have ‘two equations and two unknowns’, these equations involve the unknown constants in a manner we may not be used to. But it's still not so hard to solve for $c,k$: dividing the first equation by the second and using properties of the exponential function, the $c$ on the right side cancels, and we get $${y_1\over y_2}=e^{k(t_1-t_2)}$$ Taking a logarithm (base $e$, of course) we get $$\ln y_1-\ln y_2=k(t_1-t_2)$$ Dividing by $t_1-t_2$, this is $$k={\ln y_1-\ln y_2\over t_1-t_2}$$ Substituting back in order to find $c$, we first have $$y_1=ce^{{\ln y_1-\ln y_2\over t_1-t_2}t_1}$$ Taking the logarithm, we have $$\ln y_1=\ln c+{\ln y_1-\ln y_2\over t_1-t_2}t_1$$ Rearranging, this is $$\ln c=\ln y_1-{\ln y_1-\ln y_2\over t_1-t_2}t_1= {t_1\ln y_2-t_2\ln y_1\over t_1-t_2}$$ Therefore, in summary, the two equations $$y_1=ce^{kt_1}$$ $$y_2=ce^{kt_2}$$ allow us to solve for $c,k$, giving $$k={\ln y_1-\ln y_2\over t_1-t_2}$$ $$c=e^{t_1\ln y_2-t_2\ln y_1\over t_1-t_2}$$

A person might manage to remember such formulas, or it might be wiser to remember the way of deriving them.

A herd of llamas has $1000$ llamas in it, and the population is growing exponentially. At time $t=4$ it has $2000$ llamas. Write a formula for the number of llamas at arbitrary time $t$.

Solution: Here there is no direct mention of differential equations, but use of the buzz-phrase ‘growing exponentially’ must be taken as indicator that we are talking about the situation $$f(t)=ce^{kt}$$ where here $f(t)$ is the number of llamas at time $t$ and $c,k$ are constants to be determined from the information given in the problem. And the use of language should probably be taken to mean that at time $t=0$ there are $1000$ llamas, and at time $t=4$ there are $2000$. Then, either repeating the method above or plugging into the formula derived by the method, we find $$c=\hbox{ value of $f$ at $t=0$ } = 1000$$ $$k={\ln f(t_1)-\ln f(t_2)\over t_1-t_2}={\ln 1000-\ln 2000\over 0-4}$$ $$={ \ln {1000\over 2000}}{-4}={\ln {1\over 2} \over -4 } = (\ln 2)/4$$ Therefore, $$f(t)=1000\;e^{{\ln 2\over 4}t}=1000\cdot 2^{t/4}$$ This is the desired formula for the number of llamas at arbitrary time $t$.

  • A colony of bacteria is growing exponentially. At time $t=0$ it has $10$ bacteria in it, and at time $t=4$ it has $2000$. At what time will it have $100,000$ bacteria?

Solution: Even though it is not explicitly demanded, we need to find the general formula for the number $f(t)$ of bacteria at time $t$, set this expression equal to $100,000$, and solve for $t$. Again, we can take a little shortcut here since we know that $c=f(0)$ and we are given that $f(0)=10$. (This is easier than using the bulkier more general formula for finding $c$). And use the formula for $k$: $$k={\ln f(t_1)-\ln f(t_2)\over t_1-t_2}= {\ln 10 -\ln 2,000\over 0-4}={ \ln {10\over 2,000} \over -4 }= {\ln 200\over 4} $$ Therefore, we have $$f(t)=10\cdot e^{{\ln 200\over 4}\;t}=10\cdot 200^{t/4}$$ as the general formula. Now we try to solve $$100,000=10\cdot e^{{\ln 200\over 4}\;t}$$ for $t$: divide both sides by the $10$ and take logarithms, to get $$\ln 10,000={\ln 200\over 4}\;t$$ Thus, $$t=4\,{\ln 10,000\over \ln 200}\approx 6.953407835.$$

  • A herd of llamas is growing exponentially. At time $t=0$ it has $1000$ llamas in it, and at time $t=4$ it has $2000$ llamas. Write a formula for the number of llamas at arbitrary time $t$.
  • A herd of elephants is growing exponentially. At time $t=2$ it has $1000$ elephants in it, and at time $t=4$ it has $2000$ elephants. Write a formula for the number of elephants at arbitrary time $t$.
  • A colony of bacteria is growing exponentially. At time $t=2$ it has $10$ bacteria in it, and at time $t=4$ it has $2000$. At what time will it have $100,000$ bacteria?

Thread navigation

Calculus refresher.

  • Previous: L'Hospital's rule
  • Next: The second and higher derivatives

Similar pages

  • An introduction to ordinary differential equations
  • Another differential equation: projectile motion
  • Spruce budworm outbreak model
  • The Forward Euler algorithm for solving an autonomous differential equation
  • Introduction to bifurcations of a differential equation
  • Ordinary differential equation examples
  • Solving linear ordinary differential equations using an integrating factor
  • Examples of solving linear ordinary differential equations using an integrating factor
  • Solving single autonomous differential equations using graphical methods
  • Single autonomous differential equation problems
  • More similar pages

Module 5: Exponential and Logarithmic Functions

▪ exponential growth and decay, learning outcomes.

  • Graph exponential growth and decay functions.
  • Solve problems involving radioactive decay, carbon dating, and half life.

Exponential Growth and Decay

Tip for success.

As you learn about modelling exponential growth and decay, recall familiar techniques that have helped you to model situations using other types of functions. The exponential growth and decay models possess input and corresponding output like other functions graphed in the real plane. They also adhere to the characteristic feature of the graphs of all functions; each point (corresponding input and output pairs) contained on the graph of the function satisfies the equation of the function.

You can rely on these features as you work with the form of the function to fill in known quantities from a given situation to solve for unknowns.

In real-world applications, we need to model the behavior of a function. In mathematical modeling, we choose a familiar general function with properties that suggest that it will model the real-world phenomenon we wish to analyze. In the case of rapid growth, we may choose the exponential growth function:

[latex]y={A}_{0}{e}^{kt}[/latex]

where [latex]{A}_{0}[/latex] is equal to the value at time zero, e  is Euler’s constant, and k  is a positive constant that determines the rate (percentage) of growth. We may use the exponential growth function in applications involving doubling time , the time it takes for a quantity to double. Such phenomena as wildlife populations, financial investments, biological samples, and natural resources may exhibit growth based on a doubling time. In some applications, however, as we will see when we discuss the logistic equation, the logistic model sometimes fits the data better than the exponential model.

On the other hand, if a quantity is falling rapidly toward zero, without ever reaching zero, then we should probably choose the exponential decay model. Again, we have the form [latex]y={A}_{0}{e}^{-kt}[/latex] where [latex]{A}_{0}[/latex] is the starting value, and e  is Euler’s constant. Now k  is a negative constant that determines the rate of decay. We may use the exponential decay model when we are calculating half-life , or the time it takes for a substance to exponentially decay to half of its original quantity. We use half-life in applications involving radioactive isotopes.

In our choice of a function to serve as a mathematical model, we often use data points gathered by careful observation and measurement to construct points on a graph and hope we can recognize the shape of the graph. Exponential growth and decay graphs have a distinctive shape, as we can see in the graphs below. It is important to remember that, although parts of each of the two graphs seem to lie on the x -axis, they are really a tiny distance above the x -axis.

Graph of y=2e^(3x) with the labeled points (-1/3, 2/e), (0, 2), and (1/3, 2e) and with the asymptote at y=0.

A graph showing exponential growth. The equation is [latex]y=2{e}^{3x}[/latex].

Graph of y=3e^(-2x) with the labeled points (-1/2, 3e), (0, 3), and (1/2, 3/e) and with the asymptote at y=0.

A graph showing exponential decay. The equation is [latex]y=3{e}^{-2x}[/latex].

Exponential growth and decay often involve very large or very small numbers. To describe these numbers, we often use orders of magnitude. The order of magnitude is the power of ten when the number is expressed in scientific notation with one digit to the left of the decimal. For example, the distance to the nearest star, Proxima Centauri , measured in kilometers, is 40,113,497,200,000 kilometers. Expressed in scientific notation, this is [latex]4.01134972\times {10}^{13}[/latex]. We could describe this number as having order of magnitude [latex]{10}^{13}[/latex].

A General Note: Characteristics of the Exponential Function [latex]y=A_{0}e^{kt}[/latex]

An exponential function of the form [latex]y={A}_{0}{e}^{kt}[/latex] has the following characteristics:

  • one-to-one function
  • horizontal asymptote: y  = 0
  • domain: [latex]\left(-\infty , \infty \right)[/latex]
  • range: [latex]\left(0,\infty \right)[/latex]
  • x intercept: none
  • y-intercept: [latex]\left(0,{A}_{0}\right)[/latex]
  • increasing if k  > 0
  • decreasing if k  < 0

An exponential function models exponential growth when [latex]k� > 0[/latex] and exponential decay when [latex]k� < 0[/latex].

The graphs below include important characteristics of an exponential function that can illustrate whether the function represents growth or decay and the speed at which the function grows or decays.

The first graph is when k>0 and with the labeled points (1/k, (A_0)e), (0, A_0), and (-1/k, (A_0)/e). The second graph is of when k<0 and the with the labeled points (-1/k, (A_0)e), (0, A_0), and (1/k, (A_0)/e)

Your task is to add the key features from the graphs above to graph made using an online graphing calculator and adjusting the [latex]a[/latex] and [latex]k[/latex] values. The function [latex]f(x) = a\cdot{e}^{kx}[/latex] will be what you use as your starting point.

Include the following key features:

  • The horizontal asymptote
  • y-intercept
  • The key points labeled in the still graphs above

Below is what the graph would look like in Desmos, but you can use any online graphing tool.

problem solving exponential growth and decay

Work through the examples in this section on paper, perhaps more than once or twice, to obtain a thorough understanding of the processes involved.

Example: Graphing Exponential Growth

A population of bacteria doubles every hour. If the culture started with 10 bacteria, graph the population as a function of time.

When an amount grows at a fixed percent per unit time, the growth is exponential. To find [latex]{A}_{0}[/latex] we use the fact that [latex]{A}_{0}[/latex] is the amount at time zero, so [latex]{A}_{0}=10[/latex]. To find k , use the fact that after one hour [latex]\left(t=1\right)[/latex] the population doubles from 10 to 20. The formula is derived as follows:

[latex]\begin{array}{l}\text{ }20=10{e}^{k\cdot 1}\hfill & \hfill \\ \text{ }2={e}^{k}\hfill & \text{Divide both sides by 10}\hfill \\ \mathrm{ln}2=k\hfill & \text{Take the natural logarithm of both sides}\hfill \end{array}[/latex]

so [latex]k=\mathrm{ln}\left(2\right)[/latex]. Thus the equation we want to graph is [latex]y=10{e}^{\left(\mathrm{ln}2\right)t}=10{\left({e}^{\mathrm{ln}2}\right)}^{t}=10\cdot {2}^{t}[/latex]. The graph is shown below.

A graph starting at ten on the y-axis and rising rapidly to the right.

The graph of [latex]y=10{e}^{\left(\mathrm{ln}2\right)t}[/latex] .

Analysis of the Solution

The population of bacteria after ten hours is 10,240. We could describe this amount as being of the order of magnitude [latex]{10}^{4}[/latex]. The population of bacteria after twenty hours is 10,485,760 which is of the order of magnitude [latex]{10}^{7}[/latex], so we could say that the population has increased by three orders of magnitude in ten hours.

Calculating Doubling Time

For growing quantities, we might want to find out how long it takes for a quantity to double. As we mentioned above, the time it takes for a quantity to double is called the doubling time .

Given the basic exponential growth equation [latex]A={A}_{0}{e}^{kt}[/latex], doubling time can be found by solving for when the original quantity has doubled, that is, by solving [latex]2{A}_{0}={A}_{0}{e}^{kt}[/latex].

The formula is derived as follows:

[latex]\begin{array}{l}2{A}_{0}={A}_{0}{e}^{kt}\hfill & \hfill \\ 2={e}^{kt}\hfill & \text{Divide both sides by }{A}_{0}.\hfill \\ \mathrm{ln}2=kt\hfill & \text{Take the natural logarithm of both sides}.\hfill \\ t=\frac{\mathrm{ln}2}{k}\hfill & \text{Divide by the coefficient of }t.\hfill \end{array}[/latex]

Thus the doubling time is

[latex]t=\frac{\mathrm{ln}2}{k}[/latex]

Example: Finding a Function That Describes Exponential Growth

According to Moore’s Law, the doubling time for the number of transistors that can be put on a computer chip is approximately two years. Give a function that describes this behavior.

[latex]\begin{array}{l}t=\frac{\mathrm{ln}2}{k}\hfill & \text{The doubling time formula}.\hfill \\ 2=\frac{\mathrm{ln}2}{k}\hfill & \text{Use a doubling time of two years}.\hfill \\ k=\frac{\mathrm{ln}2}{2}\hfill & \text{Multiply by }k\text{ and divide by 2}.\hfill \\ A={A}_{0}{e}^{\frac{\mathrm{ln}2}{2}t}\hfill & \text{Substitute }k\text{ into the continuous growth formula}.\hfill \end{array}[/latex]

The function is [latex]A={A}_{0}{e}^{\frac{\mathrm{ln}2}{2}t}[/latex].

Recent data suggests that, as of 2013, the rate of growth predicted by Moore’s Law no longer holds. Growth has slowed to a doubling time of approximately three years. Find the new function that takes that longer doubling time into account.

[latex]f\left(t\right)={A}_{0}{e}^{\frac{\mathrm{ln}2}{3}t}[/latex]

We now turn to exponential decay . One of the common terms associated with exponential decay, as stated above, is half-life , the length of time it takes an exponentially decaying quantity to decrease to half its original amount. Every radioactive isotope has a half-life, and the process describing the exponential decay of an isotope is called radioactive decay.

To find the half-life of a function describing exponential decay, solve the following equation:

[latex]\frac{1}{2}{A}_{0}={A}_{o}{e}^{kt}[/latex]

We find that the half-life depends only on the constant k  and not on the starting quantity [latex]{A}_{0}[/latex].

The formula is derived as follows

[latex]\begin{array}{l}\frac{1}{2}{A}_{0}={A}_{o}{e}^{kt}\hfill & \hfill \\ \frac{1}{2}={e}^{kt}\hfill & \text{Divide both sides by }{A}_{0}.\hfill \\ \mathrm{ln}\left(\frac{1}{2}\right)=kt\hfill & \text{Take the natural log of both sides}.\hfill \\ -\mathrm{ln}\left(2\right)=kt\hfill & \text{Apply properties of logarithms}.\hfill \\ -\frac{\mathrm{ln}\left(2\right)}{k}=t\hfill & \text{Divide by }k.\hfill \end{array}[/latex]

Since t , the time, is positive, k  must, as expected, be negative. This gives us the half-life formula

[latex]t=-\frac{\mathrm{ln}\left(2\right)}{k}[/latex]

In previous sections, we learned the properties and rules for both exponential and logarithmic functions. We have seen that any exponential function can be written as a logarithmic function and vice versa. We have used exponents to solve logarithmic equations and logarithms to solve exponential equations. We are now ready to combine our skills to solve equations that model real-world situations, whether the unknown is in an exponent or in the argument of a logarithm.

As you learn to model real-world applications using exponential and logarithmic functions, it will be helpful to have created a reference list of the properties and rules for exponents and logarithms as well as the techniques for solving exponential and logarithmic equations. Keep this reference list nearby as you practice working through the examples. Eventually, you’ll be able to work through new applications without referring to your notes.

The applications below use exponential and logarithmic functions to model behavior in the real world.

One such application is in science, in calculating the time it takes for half of the unstable material in a sample of a radioactive substance to decay, called its half-life . The table below lists the half-life for several of the more common radioactive substances.

Substance Use Half-life
gallium-67 nuclear medicine 80 hours
cobalt-60 manufacturing 5.3 years
technetium-99m nuclear medicine 6 hours
americium-241 construction 432 years
carbon-14 archeological dating 5,715 years
uranium-235 atomic power 703,800,000 years

We can see how widely the half-lives for these substances vary. Knowing the half-life of a substance allows us to calculate the amount remaining after a specified time. We can use the formula for radioactive decay:

[latex]\begin{array}{l}A\left(t\right)={A}_{0}{e}^{\frac{\mathrm{ln}\left(0.5\right)}{T}t}\hfill \\ A\left(t\right)={A}_{0}{e}^{\mathrm{ln}\left(0.5\right)\frac{t}{T}}\hfill \\ A\left(t\right)={A}_{0}{\left({e}^{\mathrm{ln}\left(0.5\right)}\right)}^{\frac{t}{T}}\hfill \\ A\left(t\right)={A}_{0}{\left(\frac{1}{2}\right)}^{\frac{t}{T}}\hfill \end{array}[/latex]

  • [latex]{A}_{0}[/latex] is the amount initially present
  • [latex]T[/latex] is the half-life of the substance
  • [latex]t[/latex] is the time period over which the substance is studied
  • [latex]A[/latex], or [latex]A(t)[/latex], is the amount of the substance present after time [latex]t[/latex]

How To: Given the half-life, find the decay rate

  • Write [latex]A={A}_{o}{e}^{kt}[/latex].
  • Replace A  by [latex]\frac{1}{2}{A}_{0}[/latex] and replace t  by the given half-life.
  • Solve to find k . Express k  as an exact value (do not round).

Note: It is also possible to find the decay rate using [latex]k=-\frac{\mathrm{ln}\left(2\right)}{t}[/latex].

Example: Using the Formula for Radioactive Decay to Find the Quantity of a Substance

How long will it take for 10% of a 1000-gram sample of uranium-235 to decay?

[latex]\begin{array}{l}\text{ }y=\text{1000}e\frac{\mathrm{ln}\left(0.5\right)}{\text{703,800,000}}t\hfill & \hfill \\ \text{ }900=1000{e}^{\frac{\mathrm{ln}\left(0.5\right)}{\text{703,800,000}}t}\hfill & \text{After 10% decays, 900 grams are left}.\hfill \\ \text{ }0.9={e}^{\frac{\mathrm{ln}\left(0.5\right)}{\text{703,800,000}}t}\hfill & \text{Divide by 1000}.\hfill \\ \mathrm{ln}\left(0.9\right)=\mathrm{ln}\left({e}^{\frac{\mathrm{ln}\left(0.5\right)}{\text{703,800,000}}t}\right)\hfill & \text{Take ln of both sides}.\hfill \\ \mathrm{ln}\left(0.9\right)=\frac{\mathrm{ln}\left(0.5\right)}{\text{703,800,000}}t\hfill & \text{ln}\left({e}^{M}\right)=M\hfill \\ \text{}\text{}t=\text{703,800,000}\times \frac{\mathrm{ln}\left(0.9\right)}{\mathrm{ln}\left(0.5\right)}\text{years}\hfill & \text{Solve for }t.\hfill \\ \text{}\text{}t\approx \text{106,979,777 years}\hfill & \hfill \end{array}[/latex]

Ten percent of 1000 grams is 100 grams. If 100 grams decay, the amount of uranium-235 remaining is 900 grams.

How long will it take before twenty percent of our 1000-gram sample of uranium-235 has decayed?

[latex]t=703,800,000\times \frac{\mathrm{ln}\left(0.8\right)}{\mathrm{ln}\left(0.5\right)}\text{ years }\approx \text{ }226,572,993\text{ years}[/latex].

Example: Finding the Function that Describes Radioactive Decay

The half-life of carbon-14 is 5,730 years. Express the amount of carbon-14 remaining as a function of time, t .

This formula is derived as follows.

[latex]\begin{array}{l}\text{}A={A}_{0}{e}^{kt}\hfill & \text{The continuous growth formula}.\hfill \\ 0.5{A}_{0}={A}_{0}{e}^{k\cdot 5730}\hfill & \text{Substitute the half-life for }t\text{ and }0.5{A}_{0}\text{ for }f\left(t\right).\hfill \\ \text{}0.5={e}^{5730k}\hfill & \text{Divide both sides by }{A}_{0}.\hfill \\ \mathrm{ln}\left(0.5\right)=5730k\hfill & \text{Take the natural log of both sides}.\hfill \\ \text{}k=\frac{\mathrm{ln}\left(0.5\right)}{5730}\hfill & \text{Divide by the coefficient of }k.\hfill \\ \text{}A={A}_{0}{e}^{\left(\frac{\mathrm{ln}\left(0.5\right)}{5730}\right)t}\hfill & \text{Substitute for }r\text{ in the continuous growth formula}.\hfill \end{array}[/latex]

The function that describes this continuous decay is [latex]f\left(t\right)={A}_{0}{e}^{\left(\frac{\mathrm{ln}\left(0.5\right)}{5730}\right)t}[/latex]. We observe that the coefficient of t , [latex]\frac{\mathrm{ln}\left(0.5\right)}{5730}\approx -1.2097×10^{-4}[/latex] is negative, as expected in the case of exponential decay.

The half-life of plutonium-244 is 80,000,000 years. Find a function that gives the amount of plutonium-244 remaining as a function of time measured in years.

[latex]f\left(t\right)={A}_{0}{e}^{-0.0000000087t}[/latex]

Radiocarbon Dating

The formula for radioactive decay is important in radiocarbon dating  which is used to calculate the approximate date a plant or animal died. Radiocarbon dating was discovered in 1949 by Willard Libby who won a Nobel Prize for his discovery. It compares the difference between the ratio of two isotopes of carbon in an organic artifact or fossil to the ratio of those two isotopes in the air. It is believed to be accurate to within about 1% error for plants or animals that died within the last 60,000 years.

Carbon-14 is a radioactive isotope of carbon that has a half-life of 5,730 years. It occurs in small quantities in the carbon dioxide in the air we breathe. Most of the carbon on Earth is carbon-12 which has an atomic weight of 12 and is not radioactive. Scientists have determined the ratio of carbon-14 to carbon-12 in the air for the last 60,000 years using tree rings and other organic samples of known dates—although the ratio has changed slightly over the centuries.

As long as a plant or animal is alive, the ratio of the two isotopes of carbon in its body is close to the ratio in the atmosphere. When it dies, the carbon-14 in its body decays and is not replaced. By comparing the ratio of carbon-14 to carbon-12 in a decaying sample to the known ratio in the atmosphere, the date the plant or animal died can be approximated.

Since the half-life of carbon-14 is 5,730 years, the formula for the amount of carbon-14 remaining after t  years is

[latex]A\approx {A}_{0}{e}^{\left(\frac{\mathrm{ln}\left(0.5\right)}{5730}\right)t}[/latex]

  • [latex]A[/latex] is the amount of carbon-14 remaining
  • [latex]{A}_{0}[/latex] is the amount of carbon-14 when the plant or animal began decaying.

This formula is derived as follows:

[latex]\begin{array}{l}\text{ }A={A}_{0}{e}^{kt}\hfill & \text{The continuous growth formula}.\hfill \\ \text{ }0.5{A}_{0}={A}_{0}{e}^{k\cdot 5730}\hfill & \text{Substitute the half-life for }t\text{ and }0.5{A}_{0}\text{ for }f\left(t\right).\hfill \\ \text{ }0.5={e}^{5730k}\hfill & \text{Divide both sides by }{A}_{0}.\hfill \\ \mathrm{ln}\left(0.5\right)=5730k\hfill & \text{Take the natural log of both sides}.\hfill \\ \text{ }k=\frac{\mathrm{ln}\left(0.5\right)}{5730}\hfill & \text{Divide both sides by the coefficient of }k.\hfill \\ \text{ }A={A}_{0}{e}^{\left(\frac{\mathrm{ln}\left(0.5\right)}{5730}\right)t}\hfill & \text{Substitute for }r\text{ in the continuous growth formula}.\hfill \end{array}[/latex]

To find the age of an object we solve this equation for t :

[latex]t=\frac{\mathrm{ln}\left(\frac{A}{{A}_{0}}\right)}{-0.000121}[/latex]

Out of necessity, we neglect here the many details that a scientist takes into consideration when doing carbon-14 dating, and we only look at the basic formula. The ratio of carbon-14 to carbon-12 in the atmosphere is approximately 0.0000000001%. Let r  be the ratio of carbon-14 to carbon-12 in the organic artifact or fossil to be dated determined by a method called liquid scintillation. From the equation [latex]A\approx {A}_{0}{e}^{-0.000121t}[/latex] we know the ratio of the percentage of carbon-14 in the object we are dating to the percentage of carbon-14 in the atmosphere is [latex]r=\frac{A}{{A}_{0}}\approx {e}^{-0.000121t}[/latex]. We solve this equation for t , to get

[latex]t=\frac{\mathrm{ln}\left(r\right)}{-0.000121}[/latex]

How To: Given the percentage of carbon-14 in an object, determine its age

  • Express the given percentage of carbon-14 as an equivalent decimal r .
  • Substitute for r  in the equation [latex]t=\frac{\mathrm{ln}\left(r\right)}{-0.000121}[/latex] and solve for the age, t .

Example: Finding the Age of a Bone

A bone fragment is found that contains 20% of its original carbon-14. To the nearest year, how old is the bone?

We substitute 20% = 0.20 for r  in the equation and solve for t :

[latex]\begin{array}{l}t=\frac{\mathrm{ln}\left(r\right)}{-0.000121}\hfill & \text{Use the general form of the equation}.\hfill \\ =\frac{\mathrm{ln}\left(0.20\right)}{-0.000121}\hfill & \text{Substitute for }r.\hfill \\ \approx 13301\hfill & \text{Round to the nearest year}.\hfill \end{array}[/latex]

The bone fragment is about 13,301 years old.

The instruments that measure the percentage of carbon-14 are extremely sensitive and, as we mention above, a scientist will need to do much more work than we did in order to be satisfied. Even so, carbon dating is only accurate to about 1%, so this age should be given as [latex]\text{13,301 years}\pm \text{1% or 13,301 years}\pm \text{133 years}[/latex].

Cesium-137 has a half-life of about 30 years. If we begin with 200 mg of cesium-137, will it take more or less than 230 years until only 1 milligram remains?

less than 230 years; 229.3157 to be exact

  • Revision and Adaptation. Provided by : Lumen Learning. License : CC BY: Attribution
  • Exponential Growth and Decay Interactive. Authored by : Lumen Learning. Located at : https://www.desmos.com/calculator/4zae7vnjfr . License : Public Domain: No Known Copyright
  • Question ID 29686. Authored by : McClure,Caren. License : CC BY: Attribution . License Terms : IMathAS Community License CC-BY + GPL
  • College Algebra. Authored by : Abramson, Jay et al.. Provided by : OpenStax. Located at : http://cnx.org/contents/[email protected] . License : CC BY: Attribution . License Terms : Download for free at http://cnx.org/contents/[email protected]
  • Question ID 100026. Authored by : Rieman,Rick. License : CC BY: Attribution . License Terms : IMathAS Community License CC-BY + GPL

Footer Logo Lumen Candela

Privacy Policy

If you're seeing this message, it means we're having trouble loading external resources on our website.

If you're behind a web filter, please make sure that the domains *.kastatic.org and *.kasandbox.org are unblocked.

To log in and use all the features of Khan Academy, please enable JavaScript in your browser.

Class 12 Physics (India)

Course: class 12 physics (india)   >   unit 13.

  • Half-life and carbon dating
  • Half-life (qualitative)
  • Worked example: Half-life
  • Half life (intermediate)
  • Exponential decay formula proof (can skip, involves calculus)
  • Activity and Mean life

Exponential decay problem solving

  • More exponential decay examples
  • Half-life plot
  • Worked example: Fraction of undecayed nuclei
  • Exponential decay and semi-log plots
  • Atomic nucleus questions

problem solving exponential growth and decay

Want to join the conversation?

  • Upvote Button navigates to signup page
  • Downvote Button navigates to signup page
  • Flag Button navigates to signup page

Good Answer

Video transcript

Has anyone done Exponential Growth and Decay (in mathematical models with application A) on sos or odysseware and gotten a prefect score?? Pls help! I need help on the whole assignmnet but particularly this question..

Has Anyone Done Exponential Growth And Decay (in Mathematical Models With Application A) On Sos Or Odysseware

Step-by-step explanation:

Because it has a half-life of 25 days, half of it will decay after 25 days.

After [tex]25*4=100[/tex] days, there will be [tex](1/2)^{4}=1/2^{4}=1/16[/tex] of it left.

So, [tex]1000/16=\boxed{62.5}[/tex].

After [tex]25*3=75[/tex] days, there will be [tex](1/2)^{3}=1/2^{3}=1/8[/tex] of it left.

So, [tex]400/8=\boxed{50}[/tex]

After [tex]25*2=50[/tex] days, there will be [tex](1/2)^2=1/2^{2}=1/4[/tex] of it left.

So, [tex]300/4=\boxed{75}[/tex].

The only answer left is [tex]\boxed{100}[/tex] grams. You can solve it the same way you solved the previous steps.

Related Questions

One angle of an isosceles triangle is known to be 68°. What would be one of the other angles? a 44° b 55° c 66° d 112°

Isosceles triangles have two angles that are equal.  If the two equal angles are 68

We have  68, 68, and an unknown angle x

They add to 180

68+68+x = 180

3(2x+9)= -10 + 31 x equal (simplify your answer)

the answer is x = -1

very easy first open the bracket and then Bodmas

3[-2(8-13)] how to solve it

3 times - 2,times 8-13

You solve the ones in the bracket first

(the rule of BODMAS)

So 8-13 first, then

-2times 8-13,then

Then you get your answer

2a-3b for a= 1/2 and b =6

Substitute the given values into the expression

= 2 × [tex]\frac{1}{2}[/tex] - 3 × 6

What’s the answer for the 1st problem

Evaluate f(0) given the following graph

Assuming the graph is a function of x, then f(0) = 1.

Plug in 0 for x and ask yourself what is the y coordinate?

What is the denominator of 24⁄32? * a. 6⁄8 b. 8 c. 24 d. 32

Math expert please provide the product in standard form

Solve for X A) -6 C) 12 E) 9 B) 8 D) 3

In my thinking the answer is 9.

plz help me! 50 points

Graph attached

Shading should be done from 3 to 15/2 or 7.5

It should be open dot

Given the line below. Write the equation of the line, in point-slope form. Identify (x1, y1) as the point (-2, -1). Use the box provided to submit all of your calculations and final answers.

y + 1 = [tex]\frac{1}{2}[/tex] (x + 2)

The equation of a line in point- slope form is

y - y₁ = m(x - x₁)

where m is the slope and (x₁, y₁ ) a point on the line

Calculate m using the slope formula

m = [tex]\frac{y_{2}-y_{1} }{x_{2}-x_{1} }[/tex]

with (x₁, y₁ ) = (- 2, - 1 ) and (x₂, y₂ ) = (2, 1 )

m = [tex]\frac{1-(-1)}{2-(-2)}[/tex] = [tex]\frac{1+1}{2+2}[/tex] = [tex]\frac{2}{4}[/tex] = [tex]\frac{1}{2}[/tex] and (x₁, y₁ ) = (- 2, - 1 ) , then

y - (- 1) = [tex]\frac{1}{2}[/tex](x - (- 2) ) , that is

Multiply. Enter the product in simplest form. 3/5 × 2/3 =

0.4 or 4/10

-- x -- = -- = --

The second hand of the clock is 3.7 cm long. Find the linear speed of the tip of the second hand as it's passes around the clock face. O 0.00646 cm per sec O 172.03 cm per sec O 0.387 cm per sec 2.87 cm per sec

I'd look it up would probably work if not see if someone else might be able to help you out with this

A triangles angles have a ratio of 294 what are all three angles

24:108:40       in this order

Classify the system of equations and identify the number of solutions. 4x + 2y = −8 6x + 3y = 9 inconsistent; none consistent, dependent; one inconsistent; one consistent, dependent; infinite HELP ASAP PLS

inconsistent, none

Write both equations in slope-intercept form to see if the lines are parallel.

These two lines have slope m = −2 and different y-intercepts, therefore they are parallel.

Since the lines are parallel, the system is inconsistent and there are no solutions.

The system of equations  4x + 2y = −8 6x + 3y = 9 is inconsistent and has no solutions.

A system of equations is a collection of two or more equations with a same set of unknowns.

We need to write both equations in slope-intercept form to see if the lines are parallel.

y=-2x-4, m₁=-2

and 6x+3y=9

y=-2x+3, m₂=-2.

So two lines have slope m = −2 and different y-intercepts,

It means two line are parallel.

Hence 4x + 2y = −8 , 6x + 3y = 9 is inconsistent and has no solution.

To learn more on system of equations click:

https://brainly.com/question/24065247

PLZZZ HELP ASAP ON THIS I NEED IT NOW

1.) x={ -4,-2,0,2,4}

y={ -4,-2,0,2,4}

2.)x={ -4,-2,0,2,4}

y={{-5,-1,3,7,11}

in coordinates:

1.)(-4,-4) (-2,-2) (0,0) (2,2) (4,4)

2.)(-4,-5) (-2,-1) (0,3) (2,7) (4,11)

O is the centre of the circle. Determine the value of a°.

a triangle has 180º total

since O is the center that makes OQ and OP radii and as such they are equally distanced

this means that the m<a and the measure opposite it are equal

so ÷82 by 2

If £2000 is placed into a bank account that pays 3% compound interest per year, how much will be in the account after 2 years? 60%

There would be 2,120 dollars in that account

A=P(1 - r/n)^nt

A = 5,000 (1 - 0.03/1)^(1)12)

A = 5,000 (1 - 0.03)^2

A = 5,000 (0.9)^2

A = $5,314.06

Michael and Lindsey are saving money. Micheal begins with $20 and saves $5 per week. Lindsey begins with no money, but saves $10 per week. Enter the number of weeks it will take for Lindsey and Michael to save the same amount of money.

Step-by-step explanation: 4 Weeks

Find the first match in the same number of weeks.

25, 30, 35, 40, 45, 50, 55, 60

10, 20, 30, 40, 50, 60, 70, 80,

As you can see, 40 dollars for each person is four weeks.

plz mark brainly

If f(x)=2x^2-4, find f(0) and f(-3)

1st problem . x= √2, -√2 or Decimal form x= 1.41421356...

2nd problem .   x= √2/2 , -√2/2 or Decimal form X= 0.70710678...

What is 0.22 expressed as a percent? A. 220% B. 22% C. 2.2% D. 0.022%

0.22(100)=22.0

we have to move the decimal point twice to the right to get a percent!

0.022 i think

PLS FINDD THIS THIS IS DUE IN 10 MINNNNNN!!! PLEASE DO WITH CLEAR STEPSS!!! I'LL MARK U BRAINLIEST

There is no question to answer.

Where is the question ?

Plz help ASAP <3 james draws a line that has an x-intercept of "-5" and y-intercept of 3 on a piece of graph paper. what is the equation of the line? A) y= 5/3 x + 3 B) y=5/3 x - 5 C) y=3/5 x - 5 D) y= 3/5 x + 3

the answer is D

yeah-ya..... right?

The equation of line will be;

⇒ y = - 3/5x + 3

What is Equation of line?

The equation of line in point-slope form passing through the points

(x₁ , y₁) and (x₂, y₂) with slope m is defined as;

⇒ y - y₁ = m (x - x₁)

Where, m = (y₂ - y₁) / (x₂ - x₁)

Given that;

James draws a line that has an x-intercept of "-5" and y-intercept of 3 on a piece of graph paper .

Since, James draws a line that has an x-intercept of "-5" and y-intercept of 3 on a piece of graph paper.

Hence, The slope of the line = - 3/5

And, The y - intercept = 3

Thus, The equation of line will be;

Learn more about the equation of line visit:

https://brainly.com/question/18831322

What is an algebraic expression for this word phrase? 5 more than the product of 7 and n 5 + 7n 5(7+n) 7(5+n) 5 + 7 + n

The value of the algebraic expression when n is given as 6 will be 38.

An algebraic expression is when we use numbers and words in solving a particular mathematical question.

Based on the information given in the question, the expression will be:

= 1 + 7n - 5

Note that the value of n = 6

Therefore, 1 + 7n - 5 will be:

= 1 + 7(6) - 5

= 1 + 42 - 5

Step-by-step explanation:Therefore, the value of the algebraic expression when n is given as 6 is 38.

For her phone service, Yolanda pays a monthly fee of $12, and she pays an additional $0.05 per minute of use. The least she has been charged in a month is $63.80 What are the possible number of minutes she has used her phone in a month? Use m for the number of minutes, and solve your inequality for m.

12 + 0.05m = 63.80

12 + 1m/20 = 63.8

12 + m/20 = 63.8

(20 * 12)/20 + m/20 = 63.8

(20 * 12 + m)/20 = 63.8

(m + 240)/20 = 63.8

20((m + 240)/20) = 63.8

m + 240 = 20 * 63.8

m + 240 = 1276

m = 1276 - 240

What is the result of gaining 10 pound then losing 20

-10 pounds or 10 pounds loss

you subreact 10-20 and you get -10

Find the slope of the following graph. -3 3 1/3

its 1/3, 1 to right and 3 up

what is the measure of F? pleaseeee

The 3 angles in a triangle sum to 180°

Sum the 3 given angles and equate to 180

2x + 3 + 7x - 5 + 3x + 14 = 180 , collect like terms

12x + 12 = 180 ( subtract 12 from both sides )

12x = 168 ( divide both sides by 12 )

∠ F = 7x - 5 = 7(14) - 5 = 98 - 5 = 93° → D

You save $35 a week for a year. How much do you have at the end of the year? (Hint: Use 52 weeks please help

2. Walter bought a 21-pound bag of flour. He used 4.2 pounds of the flour. He stored the rest of the flour in 2 2/5-pound portions. What is the number of 2 2/5-pound portions he stored?

IMAGES

  1. Exponential Growth & Decay Word Problems

    problem solving exponential growth and decay

  2. Exponential Growth and Decay (examples, solutions, videos, worksheets

    problem solving exponential growth and decay

  3. Solving exponential decay problems with logarithms 2

    problem solving exponential growth and decay

  4. Proof of the Exponential Growth and Decay Equation

    problem solving exponential growth and decay

  5. Exponential Growth And Decay Word Problems Worksheet Answers

    problem solving exponential growth and decay

  6. solving exponential growth and decay problems pdf

    problem solving exponential growth and decay

VIDEO

  1. Exponential Growth and Decay| IGCSE

  2. 6.1B Exponential Growth & Decay

  3. Modeling with Exponential Functions

  4. Modeling Exponential Growth and Decay

  5. 1_1 Exponential Growth and Decay.flv

  6. Exponential Growth/Decay Functions

COMMENTS

  1. Exponential growth & decay

    Solving equations & inequalities. Unit 3. Working with units. Unit 4. Linear equations & graphs. ... Exponential expressions word problems (algebraic) Get 3 of 4 questions to level up! ... Graphing exponential growth & decay Get 3 of 4 questions to level up!

  2. Exponential growth & decay word problems (video)

    6 years ago. You can do an exponential equation without a table and going straight to the equation, Y=C (1+/- r)^T with C being the starting value, the + being for a growth problem, the - being for a decay problem, the r being the percent increase or decrease, and the T being the time.

  3. 6.8: Exponential Growth and Decay

    Key Concepts. Exponential growth and exponential decay are two of the most common applications of exponential functions. Systems that exhibit exponential growth follow a model of the form y = y0ekt. In exponential growth, the rate of growth is proportional to the quantity present. In other words, y′ = ky.

  4. Exponential Growth and Decay Word Problems

    In this section, we are going to see how to solve word problems on exponential growth and decay. Before look at the problems, if you like to learn about exponential growth and decay, please click here. Problem 1 : David owns a chain of fast food restaurants that operated 200 stores in 1999.

  5. Exponential Growth and Decay

    Growth and Decay. But sometimes things can grow (or the opposite: decay) exponentially, at least for a while. So we have a generally useful formula: y (t) = a × e kt. Where y (t) = value at time "t". a = value at the start. k = rate of growth (when >0) or decay (when <0) t = time. Example: 2 months ago you had 3 mice, you now have 18.

  6. 6.8 Exponential Growth and Decay

    Learning Objectives. 6.8.1 Use the exponential growth model in applications, including population growth and compound interest.; 6.8.2 Explain the concept of doubling time.; 6.8.3 Use the exponential decay model in applications, including radioactive decay and Newton's law of cooling.; 6.8.4 Explain the concept of half-life.

  7. PDF Exponential Growth and Decay

    Solve each exponential growth/decay problem. 1) For a period of time, an island's population grows at a rate proportional to its population. If the growth rate is 3.8% per year and the current population is 1543, what will the population be 5.2 years from now? 2) During the exponential phase, E. coli bacteria in a culture increase in number at a

  8. 8.7: Exponential Growth and Decay

    P(t) = P0e − 0.05t. The initial size is P0 grams, so we want to know when the future value P (t) at some time t will equal one-half the initial amount, P0 2. Therefore, we need to solve the equation P(t) = P0 2 for time t, which leads to the exponential equation. P0 2 = P0e − 0.05t.

  9. Exponential Growth and Decay

    Exponential Growth and Decay. These lessons, with videos, examples and step-by-step solutions, help A Level Maths students learn how to solve exponential growth and decay word problems. The following diagram shows the exponential growth and decay formula. Scroll down the page for more examples and solutions that use the exponential growth and ...

  10. Exponential Growth and Decay

    A graph showing exponential decay. The equation is y=3e−2x y = 3 e − 2 x. Exponential growth and decay often involve very large or very small numbers. To describe these numbers, we often use orders of magnitude. The order of magnitude is the power of ten when the number is expressed in scientific notation with one digit to the left of the ...

  11. Exponential Functions

    An exponential function is a function of the form f (x)=a \cdot b^x, f (x) = a⋅bx, where a a and b b are real numbers and b b is positive. Exponential functions are used to model relationships with exponential growth or decay. Exponential growth occurs when a function's rate of change is proportional to the function's current value.

  12. Exponential Growth and Decay Word Problems & Functions

    This algebra and precalculus video tutorial explains how to solve exponential growth and decay word problems. It provides the formulas and equations / funct...

  13. Exponential Growth and Decay

    Exponential decay refers to an amount of substance decreasing exponentially. Exponential decay is a type of exponential function where instead of having a variable in the base of the function, it is in the exponent. Exponential decay and exponential growth are used in carbon dating and other real-life applications. Show Step-by-step Solutions.

  14. What is exponential growth? How does it work?

    To solve exponential growth word problems, you may be plugging one value into the exponential-growth equation, and solving for the required result. But you may need to solve two problems in one, where you use, say, the doubling-time information to find the growth constant (probably by solving the exponential equation by using logarithms), and ...

  15. Solving Exponential Growth and Decay Problems

    As discussed in Exponential Growth and Decay: Introduction , all exponential growth/decay problems can be modeled using $\,P (t) = P_ {0} {\text {e}}^ {rt}\,,$ where $\,r\,$ is the relative growth rate. In the year $\,2000\,,$ the population will be about $\,1 {,}775 {,}954\,$ people. Be careful with units!

  16. Exponential Growth and Decay

    Exponential Growth: y = a (1 + r) x. Exponential Decay: y = a (1 - r) x. Remember that the original exponential formula was y = abx. You will notice that in these new growth and decay functions, the b value ( growth factor) has been replaced either by (1 + r) or by (1 - r ). The growth "rate" ( r) is determined as b = 1 + r.

  17. Exponential Growth And Decay

    Compound internet, decpreciation, growth of virus, spoiling of perishable foods, are some of the real life examples which need to be calculated using a math formula. The exponential growth and decay gives the required needed calculations using the formulas f (x) = a (1 + r) t, and f (x) = a (1 - r) t.

  18. Exponential growth and decay: a differential equation

    This little section is a tiny introduction to a very important subject and bunch of ideas: solving differential equations. We'll just look at the simplest possible example of this. The general idea is that, instead of solving equations to find unknown numbers, we might solve equations to find unknown functions. There are many possibilities for ...

  19. Exponential Growth and Decay

    The equation is [Math Processing Error] y = 2 e 3 x. A graph showing exponential decay. The equation is [Math Processing Error] y = 3 e − 2 x. Exponential growth and decay often involve very large or very small numbers. To describe these numbers, we often use orders of magnitude. The order of magnitude is the power of ten when the number is ...

  20. 11: Differential equations for exponential growth and decay

    In Section 10.2 10.2 we made an observation about exponential functions and a new kind of equation - a differential equation - that such functions satisfy. In this chapter we explore this observation in more detail. At first, this link is based on the simple relationship between an exponential function and its derivatives.

  21. Exponential decay problem solving (video)

    Exponential decay problem solving. Exponential decay refers to a process in which a quantity decreases over time, with the rate of decrease becoming proportionally smaller as the quantity gets smaller. Use the exponential decay formula to calculate k, calculating the mass of carbon-14 remaining after a given time, and calculating the time it ...

  22. Has Anyone Done Exponential Growth And Decay (in Mathematical Models

    The system of equations 4x + 2y = −8 6x + 3y = 9 is inconsistent and has no solutions.. What is system of Equations? A system of equations is a collection of two or more equations with a same set of unknowns.. We need to write both equations in slope-intercept form to see if the lines are parallel.. 4x+2y=-8. 2y=-8-4x. y=-4-2x, y=-2x-4, m₁=-2. and 6x+3y=9